SlideShare una empresa de Scribd logo
1 de 49
Combinations and Permutations
                    What's the Difference?
 In English we use the word "combination" loosely, without
 thinking if the order of things is important. In other words:

      "My fruit salad is a combination of apples, grapes and
      bananas" We don't care what order the fruits are in,
      they could also be "bananas, grapes and apples" or
      "grapes, apples and bananas", its the same fruit salad.

      "The combination to the safe was 472". Now we do
      care about the order. "724" would not work, nor would
      "247". It has to be exactly 4-7-2.

      So, in Mathematics we use more precise language:

          If the order doesn't matter, it is a Combination.
          If the order does matter it is a Permutation.

          A Permutation is an ordered Combination.
Permutation : Permutation means arrangement of things. The
word arrangement is used, if the order of things is considered.

Combination: Combination means selection of things. The word
selection is used, when the order of things has no importance.

Definition:

Permutation:
      An arrangement is called a Permutation. It is the
rearrangement of objects or symbols into distinguishable sequences.
When we set things in order, we say we have made an arrangement.
When we change the order, we say we have changed the
arrangement. So each of the arrangement that can be made by
taking some or all of a number of things is known as Permutation.

Combination:
      A Combination is a selection of some or all of a number of
different objects. It is an un-ordered collection of unique sizes.In a
permutation the order of occurence of the objects or the
arrangement is important but in combination the order of occurence
of the objects is not important.


Formula:

Permutation = nPr = n! / (n-r)!
Combination = nCr = nPr / r!
where,
       n, r are non negative integers and 0≤ r≤n.
       r is the size of each permutation.
       n is the size of the set from which elements are permuted.
       ! is the factorial operator.


Example: Suppose we have to form a number of consisting
of three digits using the digits 1,2,3,4, To form this number the
digits have to be arranged. Different numbers will get formed
depending upon the order in which we arrange the digits. This
is an example of Permutation.

Now suppose that we have to make a team of 11 players out of
20 players, This is an example of combination, because the
order of players in the team will not result in a change in the
team. No matter in which order we list out the players the team
will remain the same! For a different team to be formed at least
one player will have to be changed.

Now let us look at two fundamental principles of counting:
Addition rule : If an experiment can be performed in „n‟ ways, &
another experiment can be performed in „m‟ ways then either of
the two experiments can be performed in (m+n) ways. This rule
can be extended to any finite number of experiments.

Example:       Suppose there are 3 doors in a room, 2 on one
side and 1 on other side. A man wants to go out from the room.
Obviously he has „3‟ options for it. He can come out by door „A‟
or door „B‟ or door ‟C‟.




Multiplication Rule : If a work can be done in m ways, another
work can be done in „n‟ ways, then both of the operations can
be performed in m x n ways. It can be extended to any finite
number of operations.

Example.:    Suppose a man wants to cross-out a room, which
has 2 doors on one side and 1 door on other site. He has 2 x
1 = 2 ways for it.




Factorial n : The product of first „n‟ natural numbers is denoted
by n!.

       n! = n(n-1) (n-2) ………………..3.2.1.

       Ex.     5! = 5 x 4 x 3 x 2 x 1 =120

       Note      0!   = 1
Proof n! =n, (n-1)!

          Or         (n-1)! = [n x (n-1)!]/n = n! /n

          Putting n = 1, we have

          O! = 1!/1 = 1



Permutation

  Number of permutations of „n‟ different things taken „r‟ at a
     time is given by:-
     n
         Pr     =         n!/(n-r)!

Proof:        Say we have „n‟ different things a1, a2……, an.

Clearly the first place can be filled up in „n‟ ways. Number of
      things left after filling-up the first place = n-1

So the second-place can be filled-up in (n-1) ways. Now
      number of things left after filling-up the first and second
      places = n - 2

Now the third place can be filled-up in (n-2) ways.

Thus number of ways of filling-up first-place = n

Number of ways of filling-up second-place = n-1

Number of ways of filling-up third-place = n-2

Number of ways of filling-up r-th place = n – (r-1) = n-r+1

By multiplication – rule of counting, total no. of ways of filling
    up, first, second -- rth-place together :-
n (n-1) (n-2) ------------ (n-r+1)

         Hence:
n
    Pr    = n (n-1)(n-2) --------------(n-r+1)

= [n(n-1)(n-2)----------(n-r+1)] [(n-r)(n-r-1)-----3.2.1.] / [(n-r)(n-r-
1)] ----3.2.1

         n
             Pr = n!/(n-r)!

     Number of permutations of „n‟ different things taken all at a
        time is given by:-
              n
                  Pn              =        n!

Proof :
Now we have „n‟ objects, and n-places.

Number of ways of filling-up first-place = n

Number of ways of filling-up second-place = n-1

Number of ways of filling-up third-place = n-2

Number of ways of filling-up r-th place, i.e. last place =1

Number of ways of filling-up first, second, --- n th place
= n (n-1) (n-2) ------ 2.1.
         n
             Pn = n!

Concept.

                       n
We have                    Pr =   n!/n-r

Putting r = n, we have :-
n
           Pr = n! / (n-r)

               n
       But         Pn = n!

Clearly it is possible, only when n! = 1

Hence it is proof that       0! = 1

Note : Factorial of negative-number is not defined. The
expression –3! has no meaning.

Examples

Q. How many different signals can be made by 5 flags from 8-
flags of different colours?

Ans.        Number of ways taking 5 flags out of 8-flage = 8P5

= 8!/(8-5)!

= 8 x 7 x 6 x 5 x 4 = 6720

Q. How many words can be made by using the letters of the
    word “SIMPLETON” taken all at a time?

Ans. There are „9‟ different letters of the word “SIMPLETON”

Number of Permutations taking all the letters at a time = 9P9

= 9!        = 362880.

Number of permutations of n-thing, taken all at a time, in which
   „P‟ are of one type, „q‟ of them are of second-type, „r‟ of
   them are of third-type, and rest are all different is given by
   :-

 n!/(p! x q! x
r!)
Example: In how many ways can the letters of the word “Pre-
    University” be arranged?

13!/(2! X 2! X 2!)

Number of permutations of n-things, taken „r‟ at a time when
   each thing can be repeated r-times is given by = nr.

Proof.

Number of ways of filling-up first –place = n

Since repetition is allowed, so

         Number of ways of filling-up second-place = n

Number of ways of filling-up third-place

Number of ways of filling-up r-th place = n

Hence total number of ways in which first, second ----r th,
         places can be filled-up

= n x n x n ------------- r factors.

= nr

Example: John has 8 friends. In how many ways can he invite
one or more of them to dinner?

Ans.     John can select one or more than one of his 8 friends.

=> Required number of ways = 28 – 1= 255.
(iv) Number of ways of selecting zero or more things from „n‟
      identical things is given by :- n+1

Example:      In how many ways, can zero or more letters be
      selected form the letters AAAAA?

Ans. Number of ways of :

       Selecting zero 'A's = 1

       Selecting one 'A's = 1

       Selecting two 'A's     =1

       Selecting three 'A's = 1

       Selecting four 'A's = 1

       Selecting five 'A's    = 1

=> Required number of ways = 6              [5+1]

(V)     Number of ways of selecting one or more things from „p‟
      identical things of one type „q‟ identical things of another
      type, „r‟ identical things of the third type and „n‟ different
      things is given by :-

(p+1) (q+1) (r+1)2n – 1
Example:        Find the number of different choices that can be
       made from 3 apples, 4 bananas and 5 mangoes, if at least
       one fruit is to be chosen.

Ans:

Number of ways of selecting apples = (3+1) = 4 ways.

Number of ways of selecting bananas = (4+1) = 5 ways.

Number of ways of selecting mangoes = (5+1) = 6 ways.

Total number of ways of selecting fruits = 4 x 5 x 6

But this includes, when no fruits i.e. zero fruits is selected

=> Number of ways of selecting at least one fruit = (4x5x6) -1 =
       119

Note :- There was no fruit of a different type, hence here n=o

=> 2n = 20=1

(VI)     Number of ways of selecting „r‟ things from „n‟ identical
       things is „1‟.

Example: In how many ways 5 balls can be selected from „12‟
   identical red balls?

Ans. The balls are identical, total number of ways of selecting 5
   balls = 1.

Example: How many numbers of four digits can be formed with
   digits 1, 2, 3, 4 and 5?
Ans. Here n = 5                   [Number of digits]

And r = 4                    [ Number of places to be filled-up]

                        5
Required number is          P4 = 5!/1! = 5 x 4 x 3 x 2 x 1




Example:   A child has 3 pocket and 4 coins. In how many
    ways can he put the coins in his pocket.

Ans.     First coin can be put in 3 ways, similarly second, third
       and forth coins also can be put in 3 ways.

So total number of ways = 3 x 3 x 3 x 3 = 34 = 81




                      So, we should really call this a "Permutation
                      Lock"!


                               Permutations
             There are basically two types of permutation:

     1. Repetition is Allowed: such as the lock above. It could be
                                  "333".
   2. No Repetition: for example the first three people in a running
                   race. You can't be first and second.
1. Permutations with Repetition

                 These are the easiest to calculate.

When you have n things to choose from ... you have n choices each
                             time!

          When choosing r of them, the permutations are:

                         n × n × ... (r times)

 (In other words, there are n possibilities for the first choice, THEN
there are n possibilites for the second choice, and so on, multplying
                               each time.)

       Which is easier to write down using an exponent of r:

                      n × n × ... (r times) = nr

 Example: in the lock above, there are 10 numbers to choose from
               (0,1,..9) and you choose 3 of them:

        10 × 10 × ... (3 times) = 103 = 1,000 permutations

                     So, the formula is simply:

                                  nr

                     where n is the number of
                    things to choose from, and
                       you choose r of them
                    (Repetition allowed, order
                              matters)
2. Permutations without Repetition

In this case, you have to reduce the number of available choices each
                                time.

                                    For example, what order could 16
                                    pool balls be in?

                                    After choosing, say, number "14"
                                    you can't choose it again.

So, your first choice would have 16 possibilites, and your next choice
   would then have 15 possibilities, then 14, 13, etc. And the total
                       permutations would be:

            16 × 15 × 14 × 13 × ... = 20,922,789,888,000

But maybe you don't want to choose them all, just 3 of them, so that
                        would be only:

                        16 × 15 × 14 = 3,360

In other words, there are 3,360 different ways that 3 pool balls could
                     be selected out of 16 balls.

   But how do we write that mathematically? Answer: we use the
                       "factorial function"

              The factorial function (symbol: !) just means
              to multiply a series of descending natural
              numbers. Examples:

                    4! = 4 × 3 × 2 × 1 = 24
                    7! = 7 × 6 × 5 × 4 × 3 × 2 × 1 = 5,040
                    1! = 1

      Note: it is generally agreed that 0! = 1. It may seem
      funny that multiplying no numbers together gets you 1,
but it helps simplify a lot of equations.

 So, if you wanted to select all of the billiard balls the permutations
                              would be:

                       16! = 20,922,789,888,000

     But if you wanted to select just 3, then you have to stop the
multiplying after 14. How do you do that? There is a neat trick ... you
                           divide by 13! ...

          16 × 15 × 14 × 13 × 12 ...
                                             = 16 × 15 × 14 = 3,360
                               13 × 12 ...

                 Do you see? 16! / 13! = 16 × 15 × 14

                           The formula is written:




                        where n is the number of
                       things to choose from, and
                          you choose r of them
                     (No repetition, order matters)

                                   Examples:

      Our "order of 3 out of 16 pool balls example" would be:

         16!         16!           20,922,789,888,000
                 =         =                                  = 3,360
       (16-3)!       13!              6,227,020,800

          (which is just the same as: 16 × 15 × 14 = 3,360)
How many ways can first and second place be awarded to 10 people?

            10!              10!          3,628,800
                        =           =                        = 90
          (10-2)!             8!              40,320


                  (which is just the same as: 10 × 9 = 90)

                                   Notation

 Instead of writing the whole formula, people use different notations
                            such as these:




                            Example: P(10,2) = 90

                               Combinations

 There are also two types of combinations (remember the order does
                          not matter now):

          1. Repetition is Allowed: such as coins in your pocket
                                (5,5,5,10,10)
      2. No Repetition: such as lottery numbers (2,14,15,27,30,33)



How many permutations of 3 different digits are there, chosen from
the ten digits 0 to 9 inclusive?


A
84
B
120
C
504
D
 720
Answer :The number of permutations of 3 digits chosen from 10 is
10
   P3 = 10 × 9 × 8 = 720
Jones is the Chairman of a committee. In how many ways can a
committee of 5 be chosen from 10 people given that Jones must be
one of them?


A
126
B
252
C
495
D
 3,024
 Answer: Jones is already chosen, so we need to choose another 4
from 9.
In choosing a committee, order doesn't matter; so we need the
number of combinations of 4 people chosen from 9
= 9 C4
= 9!/(4!)(5!)
= (9 × 8 × 7 × 6)/(4 × 3 × 2 × 1)
= 3,024/24 = 126
A password consists of four different letters of the alphabet. How
many different possible passwords are there?
        (Using pen and paper can help you learn.)
A
426
B
456,976
C
14,950
D
 358,800
Answer: The number of permutations of 4 letters chosen from 26 is
26
   P4 = 26 × 25 × 24 × 23 = 358,800
A password consists of two letters of the alphabet followed by three
digits chosen from 0 to 9. Repeats are allowed. How many different
possible passwords are there?
 (Don't worry if you get it wrong ... you can learn from
                     your mistakes.)
A
492,804
B
650,000
C
676,000
D
1,757,600
Answer: The number of ways of choosing the letters = 26 × 26 = 676
The number of ways of choosing the digits = 10 × 10 × 10 = 1,000

So the number of possible passwords = 676 × 1,000 = 676,000
An encyclopedia has eight volumes. In how many ways can the eight
volumes be replaced on the shelf?
      (Use pen and paper to work out the answer.)
A
8
B
5,040
C
40,320
D
 88
 Answer:Imagine there are 8 spots on the shelf. Replace the volumes
one by one.
The first volume to be replaced could go in any one of the eight spots.
The second volume to be replaced could then go in any one of the
seven remaining spots.
The third volume to be replaced could then go in any one of the six
remaining spots.
etc

So the total number of ways the eight volumes could be replaced
= 8!
=8×7×6×5×4×3×2×1
= 40,320
Assuming that any arrangement of letters forms a 'word', how many
'words' of any length can be formed from the letters of the word
SQUARE?

(No repeating of letters)
          (No guessing! Be sure of your answer.)
A
82
B
720
C
1,956
D
9,331
Answer: The number of one letter 'words' = 6P1 = 6
The number of two letter 'words' = 6P2 = 6 × 5 = 30
The number of three letter 'words' = 6P3 = 6 × 5 × 4 = 120
The number of four letter 'words' = 6P4 = 6 × 5 × 4 × 3 = 360
The number of five letter 'words' = 6P5 = 6 × 5 × 4 × 3 × 2 = 720
The number of six letter 'words' = 6P6 = 6! = 720

So the total number of possible 'words' = 6 + 30 + 120 + 360 + 720 +
720 = 1,956
16 teams enter a competition. They are divided up into four Pools (A,
B, C and D) of four teams each.

Every team plays one match against the other teams in its Pool.

After the Pool matches are completed:
• the winner of Pool A plays the second placed team of Pool B
• the winner of Pool B plays the second placed team of Pool A
• the winner of Pool C plays the second placed team of Pool D
• the winner of Pool D plays the second placed team of Pool C

The winners of these four matches then play semi-finals, and the
winners of the semi-finals play in the final.

How many matches are played altogether?
        (Using pen and paper can help you learn.)
A
23
B
31
C
32
D
63
Answer:The number of matches played in each Pool = 4C2 = 4!/(2!2!)
= (4 × 3)/(2 × 1) = 6
So the total number of Pool matches = 4 × 6 = 24

The winners and second placed teams play a further 4 matches.
Then there are 2 semi-finals and 1 final

So the total number of matches = 24 + 4 + 2 + 1 = 31
A restaurant offers 5 choices of appetizer, 10 choices of main meal
and 4 choices of dessert. A customer can choose to eat just one
course, or two different courses, or all three courses. Assuming all
choices are available, how many different possible meals does the
restaurant offer?
A
329
B
310
C
200
D
19
Answer; A person who eats only an appetizer has 5 choices.
A person who eats only a main meal has 10 choices.
A person who eats only a dessert has 4 choices.

A person who eats an appetizer and a main meal has 5 × 10 = 50
choices.
A person who eats an appetizer and a dessert has 5 × 4 = 20 choices.
A person who eats a main meal and a dessert has 10 × 4 = 40 choices.

A person who eats all three courses has 5 × 10 × 4 = 200 choices

So the total number of possible meals = 5 + 10 + 4 + 50 + 20 + 40 +
200 = 329
Question:
How many ways can 4 prizes be given away to 3 boys, if each
boy is eligible for all the prizes?


     (1)256
     (2)12
     (3)81
     (4) None of these
Correct Answer - (3)


Solution:


Any one prize can be given to any one of the 3 boys and hence
there are 3 ways of distributing each prize.

Hence, the 4 prizes can be distributed in 34= 81 ways.

Question:

How many words of 4 consonants and 3 vowels can be made
from 12 consonants and 4 vowels, if all the letters are
different?


     (1)16C7.7!
     (2)12C4.4C3.7!
(3)12C3.4C4
     (4) 12C4 . 4C3
Correct Answer - (2)


Solution:
                                             12
4 consonants out of 12 can be selected in     C4 ways.

3 vowels can be selected in 4C3 ways.

Therefore, total number of groups each containing 4
consonants and 3 vowels = 12C4 . 4C3

Each group contains 7 letters, which can be arranging in 7!
ways.
                                        12
Therefore required number of words =     4 . 4C3 . 7!

Question:

In how many ways can the letters of the word EDUCATION be
rearranged so that the relative position of the vowels and
consonants remain the same as in the word EDUCATION?


     (1)9!/4
     (2)9!/(4!.5!)
     (3)4!.5!
     (4) None of these
Correct Answer - (3)
Solution:


The word EDUCATION is a 9 letter word, with none of the
letters repeating.

The vowels occupy 3, 5, 7th and 8th position in the word and
the remaining 5 positions are occupied by consonants

As the relative position of the vowels and consonants in any
arrangement should remain the same as in the word
EDUCATION, the vowels can occupy only the aforementioned 4
places and the consonants can occupy 1st, 2nd, 4th, 6th and 9th
positions.

The 4 vowels can be arranged in the 3rd, 5th, 7th and 8th
position in 4! Ways.

Similarly, the 5 consonants can be arranged in 1st, 2nd, 4th, 6th
and 9th position in 5! Ways.

Hence, the total number of ways = 4! . 5!.
Question:
There are 12 yes or no questions. How many ways can these
be answered?


     (1)1024
     (2)2048
     (3) 4096
     (4) 144
Correct Answer - (3)
Solution:


Each of the questions can be answered in 2 ways (yes or no)

Therefore, no. of ways of answering 12 questions = 212 = 4096
ways.
Question:
What is the value of 1.1! + 2.2! + 3!.3! + ............ n.n!,
where n! means n factorial or n(n-1)(n-2)...1
Solution:

1.1! = (2 -1).1! = 2.1! – 1.1! = 2! - 1!
2.2! = (3 - 1).2! = 3.2! - 2! = 3! - 2!
3.3! = (4 - 1).3! = 4.3! - 3! = 4! - 3!
..
..
..
n.n! = (n+1 - 1).n! = (n+1)(n!) - n! = (n+1)! - n!

Summing up all these terms, we get (n+1)! - 1!
Question:
In how many ways can the letters of the word MANAGEMENT
be rearranged so that the two As do not appear together?


Solution:

The word MANAGEMENT is a 10 letter word.

Normally, any 10 letter word can be rearranged in 10! ways.

However, as there are certain letters of the word repeating,
we need to account for those. In this case, the letters A, M, E
and N repeat twice each.
Therefore, the number of ways in which the letters of the
word MANAGEMENT can be rearranged reduces to                .

The problem requires us to find out the number of outcomes
in which the two As do not appear together.

The number of outcomes in which the two As appear together
can be found out by considering the two As as one single
letter. Therefore, there will now be only 9 letters of which
three of them E, N and M repeat twice. So these 9 letters with
3 of them repeating twice can be rearranged in        ways.

Therefore, the required answer in which the two As do not
appear next to each other =

Total number of outcomes - the number of outcomes in which
the 2 As appear together


=>                       ways.
**Question:
There are 5 Rock songs, 6 Carnatic songs and 3 Indi pop songs.
How many different albums can be formed using the above
repertoire if the albums should contain at least 1 Rock song
and 1 Carnatic song?


    (1)15624
    (2)16384
    (3)6144
    (4) 240
Correct Answer - (1)


Solution:

There are 2n ways of choosing ‘n’ objects. For e.g. if n = 3,
then the three objects can be chosen in the following 23 ways
- 3C0 ways of choosing none of the three, 3C1 ways of choosing
one out of the three, 3C2 ways of choosing two out of the
three and 3C3 ways of choosing all three.

In the given problem, there are 5 Rock songs. We can choose
them in 25 ways. However, as the problem states that the
case where you do not choose a Rock song does not exist (at
least one rock song has to be selected), it can be done in 25 -
1 = 32 - 1 = 31 ways.

Similarly, the 6 Carnatic songs, choosing at least one, can be
selected in 26 - 1 = 64 - 1 = 63 ways.

And the 3 Indi pop can be selected in 23 = 8 ways. Here the
option of not selecting even one Indi Pop is allowed.

Therefore, the total number of combinations = 31 . 63 . 8 =
15624
Question:
How many words can be formed by re-arranging the letters of
the word ASCENT such that A and T occupy the first and last
position respectively?


     (1)5!
     (2)4!
(3)2!
     (4) 6! / 2!
Correct Answer - (2)
Solution:


As A and T should occupy the first and last position, the first
and last position can be filled in only one way. The remaining
4 positions can be filled in 4! Ways by the remaining words
(S,C,E,N,T). hence by rearranging the letters of the word
ASCENT we can form 1x4! = 4! words.
**Question:
A team of 8 students goes on an excursion, in two cars, of
which one can seat 5 and the other only 4. In how many ways
can they travel?


Solution:


There are 8 students and the maximum capacity of the cars
together is 9.

We may divide the 8 students as follows

Case I: 5 students in the first car and 3 in the second

Or Case II: 4 students in the first car and 4 in the second

Hence, in Case I: 8 students are divided into groups of 5 and 3
in 8C3 ways.

Similarly, in Case II: 8 students are divided into two groups of
4 and 4 in 8C4 ways.
Therefore, the total number of ways in which 8 students can
travel is 8C3 + 8C4 = 56 + 70 = 126.
**Question:
When four fair dice are rolled simultaneously, in how many
outcomes will at least one of the dice show 3?


Solution:


When 4 dice are rolled simultaneously, there will be a total of
64 = 1296 outcomes.

The number of outcomes in which none of the 4 dice show 3
will be 54 = 625 outcomes.

Therefore, the number of outcomes in which at least one die
will show 3 = 1296 – 625 = 671

Question:
How many ways can 10 letters be posted in 5 post boxes, if
each of the post boxes can take more than 10 letters?
Solution:

Each of the 10 letters can be posted in any of the 5 boxes.

So, the first letter has 5 options, so does the second letter
and so on and so forth for all of the 10 letters.

i.e. 5.5.5….5 (upto 10 times)

= 510.

Here is another way to calculate it:
Including "none" as an option, there are 6 choices of appetizer, 11
choices of main meal and 5 choices of dessert. Thus the total number
of choices is 6x11x5=330.

One of these is not a meal though (no appetizer, no main meal and no
dessert), so there are 329 possible meals.

Circular permutations

There are two cases of circular-permutations:-

(a)     If clockwise and anti clock-wise orders are different,
      then total number of circular-permutations is given by (n-
      1)!

(b)     If clock-wise and anti-clock-wise orders are taken as not
      different, then total number of circular-permutations is
      given by (n-1)!/2!

Proof(a):




(a)     Let‟s consider that 4 persons A,B,C, and D are sitting
      around a round table

Shifting A, B, C, D, one position in anticlock-wise direction, we
get the following agreements:-
Thus, we use that if 4 persons are sitting at a round table, then
they can be shifted four times, but these four arrangements will
be the same, because the sequence of A, B, C, D, is same. But
if A, B, C, D, are sitting in a row, and they are shifted, then the
four linear-arrangement will be different.




Hence if we have „4‟ things, then for each circular-arrangement
number of linear-arrangements =4

Similarly, if we have „n‟ things, then for each circular –
agreement, number of linear – arrangement = n.

Let the total circular arrangement = p

Total number of linear–arrangements = n.p

Total number of linear–arrangements

= n. (number of circular-arrangements)

Or Number of circular-arrangements = 1 (number of linear
arrangements)

n = 1( n!)/n

circular permutation = (n-1)!

Proof (b) When clock-wise and anti-clock wise arrangements
       are not different, then observation can be made from
       both sides, and this will be the same. Here two
       permutations will be counted as one. So total
       permutations will be half, hence in this case.

     Circular–permutations = (n-1)!/2
Note: Number of circular-permutations of „n‟ different things
taken „r‟ at a time:-

   (a) If clock-wise and anti-clockwise orders are taken as
         different, then total number of circular-permutations =
         n
           Pr /r

   (b) If clock-wise and anti-clockwise orders are taken as not
         different, then total number of circular – permutation =
             n
               Pr/2r

Example: How many necklace of 12 beads each can be made
    from 18 beads of different colours?

Ans. Here clock-wise and anti-clockwise arrangement s are
same.
                                                    18
Hence total number of circular–permutations:             P12/2x12

= 18!/(6       x 24)

                       Restricted – Permutations

(a) Number of permutations of „n‟ things, taken „r‟ at a time,
    when a particular thing is to be always included in each
    arrangement

= r n-1 Pr-1

(b) Number of permutations of „n‟ things, taken „r‟ at a time,
    when a particular thing is fixed: = n-1 Pr-1

(c) Number of permutations of „n‟ things, taken „r‟ at a time,
    when a particular thing is never taken: = n-1 Pr.
(d) Number of permutations of „n‟ things, taken „r‟ at a time,
        when „m‟ specified things always come together = m! x ( n-
        m+1) !

(e) Number of permutations of „n‟ things, taken all at a time,
        when „m‟ specified things always come together = n ! - [ m!
        x (n-m+1)! ]

Example: How many words can be formed with the letters of
the word „OMEGA‟ when:

      (i)     „O‟ and „A‟ occupying end places.

      (ii)    „E‟ being always in the middle

      (iii)    Vowels occupying odd-places

      (iv)       Vowels being never together.

Ans.

(i)      When „O‟ and „A‟ occupying end-places

 => M.E.G. (OA)

 Here (OA) are fixed, hence M, E, G can be arranged in 3!
ways

But (O,A) can be arranged themselves is 2! ways.

        => Total number of words = 3! x 2! = 12 ways.
ii) When „E‟ is fixed in the middle

     =>    O.M.(E), G.A.

     Hence four-letter O.M.G.A. can be arranged in 4! i.e 24
     ways.

     (iii) Three vowels (O,E,A,) can be arranged in the odd-
          places (1st, 3rd and 5th)   = 3!    ways.

     And two consonants (M,G,) can be arranged in the even-
          place            (2nd, 4th) = 2 ! ways

     => Total number of ways= 3! x 2! = 12 ways.

     (iv) Total number of words = 5! =          120!

If all the vowels come together, then we have: (O.E.A.), M,G

These can be arranged in        3!    ways.

But (O,E.A.) can be arranged themselves in 3! ways.

=> Number of ways, when vowels come-together =          3! x
3!

= 36 ways

=> Number of ways, when vowels being never-together

= 120-36       = 84 ways.
Number of Combination of „n‟ different things, taken „r‟ at a time
is given by:-

n
    Cr= n! / r ! x (n-r)!

Proof: Each combination consists of „r‟ different things, which
can be arranged among themselves in r! ways.

=> For one combination of „r‟ different things, number of
arrangements =                  r!

      For nCr combination number of arrangements:                      r   n
                                                                               Cr

                                                       n
=> Total number of permutations =                 r!       Cr ---------------(1)

But number of permutation of „n‟ different things, taken „r‟ at a
time

= nPr -------(2)

From (1) and (2) :

           n
               Pr =         r! . nCr

      or           n!/(n-r)! = r! . nCr

               n
      or           Cr   =      n!/r!x(n-r)!

Note: nCr = nCn-r

               n
      or           Cr   = n!/r!x(n-r)! and nCn-r = n!/(n-r)!x(n-(n-r))!
= n!/(n-r)!xr!

Restricted – Combinations

      (a) Number of combinations of „n‟ different things taken „r‟ at
           a time, when „p‟ particular things are always included =
           n-p
                 Cr-p.

      (b) Number of combination of „n‟ different things, taken „r‟ at
           a time, when „p‟ particular things are always to be
           excluded = n-pCr

Example:             In how many ways can a cricket-eleven be chosen
out of 15 players? if

        (i) A particular player is always chosen,

        (ii) A particular is never chosen.

Ans:

(i)       A particular player is always chosen, it means that 10
                 players are selected out of the remaining 14 players.

                                      14
=. Required number of ways =               C10 = 14C4

= 14!/4!x19! = 1365

(ii) A particular players is never chosen, it means that 11
        players are selected out of 14 players.
14
=> Required number of ways =             C11

= 14!/11!x3! = 364

(iii) Number of ways of selecting zero or more things from „n‟
different things is given by:- 2n-1

Proof: Number of ways of selecting one thing, out of n-things
  = nC1

Number of selecting two things, out of n-things =nC2

Number of ways of selecting three things, out of n-things =nC3

  Number of ways of selecting „n‟ things out of „n‟ things = nCn

=>Total number of ways of selecting one or more things out of
n different things

= nC1 + nC2 + nC3 + ------------- + nCn

  = (nC0 + nC1 + -----------------nCn) - nC0

  = 2n – 1                    [ nC0=1]

  1. Examples:
       1. Suppose we want to select two out of three boys A, B, C.
          Then, possible selections are AB, BC and CA.

           Note: AB and BA represent the same selection.

        2. All the combinations formed by a, b, c taking ab, bc, ca.
        3. The only combination that can be formed of three letters a,
           b, c taken all at a time is abc.
4. Various groups of 2 out of four persons A, B, C, D are:

                                        AB, AC, AD, BC, BD, CD.

       5. Note that ab ba are two different permutations but they
          represent the same combination.
  2. Number of Combinations:

     The number of all combinations of n things, taken r at a time is:

                     n            n!       n(n - 1)(n - 2) ... to r factors
                      Cr =                =                                 .
                             (r!)(n - r!)                r!

     Note:
                 n
       i.          Cn = 1 and nC0 = 1.
                 n
      ii.          Cr = nC(n - r)

     Examples:

            11 (11 x 10 x 9 x 8)
     i.       C4 =                 = 330.
                (4 x 3 x 2 x 1)
                                       16 x 15 x 14 16 x 15 x 14
     ii. 16C13 = 16C(16 - 13) = 16C3 =             =            = 560.
                                            3!       3x2x1




1. From a group of 7 men and 6 women, five persons are to be
   selected to form a committee so that at least 3 men are there on the
   committee. In how many ways can it be done?
   A.564                             B. 645
   C. 735                            D.756
   2. In how many different ways can the letters of the word
   'LEADING' be arranged in such a way that the vowels always
   come together?
   A.360                             B. 480
                                        5040
   C. 720                            D.
3 how many different ways can the letters of the word
 In
.'CORPORATION' be arranged so that the vowels always come
 together?
 A                                    B
   810                                  1440
 .                                    .
 C                                    D
   2880                                 50400
 .                                    .
 4 Out of 7 consonants and 4 vowels, how many words of 3
 consonants and 2 vowels can be formed?
 A                                    B
   210                                  1050
 .                                    .
 C                                    D
   25200                                21400
 .                                    .
 5. In how many ways can the letters of the word 'LEADER' be
 arranged?
 A                                    B
   72                                   144
 .                                    .
 C                                    D
   360                                  720
 .                                    .
 6 In a group of 6 boys and 4 girls, four children are to be selected. In
 how many different ways can they be selected such that at least one
 boy should be there?
 A                                    B
   159                                  194
 .                                    .
 C                                    D
   205                                  209
 .                                    .
 E
   None of these
 .
 7 How many 3-digit numbers can be formed from the digits 2, 3, 5, 6,
 7 and 9, which are divisible by 5 and none of the digits is repeated?
 A                                    B
   5                                    10
 .                                    .
 C                                    D
   15                                   20
 .                                    .
 8 In how many ways a committee, consisting of 5 men and 6 women
 can be formed from 8 men and 10 women?
 A 266                                B 5040
.                                   .
C                                   D
  11760                               86400
.                                   .
E
  None of these
.
9 A box contains 2 white balls, 3 black balls and 4 red balls. In how
many ways can 3 balls be drawn from the box, if at least one black
ball is to be included in the draw?
A                                   B
  32                                  48
.                                   .
C                                   D
  64                                  96
.                                   .
E
  None of these
.
10 In how many different ways can the letters of the word 'DETAIL'
be arranged in such a way that the vowels occupy only the odd
positions?
A                                   B
  32                                  48
.                                   .
C                                   D
  36                                  60
.                                   .
E
  120
.
11 In how many ways can a group of 5 men and 2 women be made
out of a total of 7 men and 3 women?
A                                   B
  63                                  90
.                                   .
C                                   D
  126                                 45
.                                   .
E
  135
.
12 How many 4-letter words with or without meaning, can be formed
out of the letters of the word, 'LOGARITHMS', if repetition of letters
is not allowed?
A                                   B
  40                                  400
.                                   .
C 5040                              D 2520
.                                  .
13 In how many different ways can the letters of the word
'MATHEMATICS' be arranged so that the vowels always come
together?
A                                  B
  10080                              4989600
.                                  .
C                                  D
  120960                             None of these
.                                  .
14 In how many different ways can the letters of the word 'OPTICAL'
be arranged so that the vowels always come together?
A                                  B
  120                                720
.                                  .
C                                  D
  4320                               2160
.                                  .
E
  None of these
.




Answer:1 Option D

Explanation:

We may have (3 men and 2 women) or (4 men and 1 woman) or (5
men only).

  Required number of ways= (7C3 x 6C2) + (7C4 x 6C1) + (7C5)
                            7x6x5 6x5
                         =           x       + (7C3 x 6C1) + (7C2)
                            3x2x1 2x1
                                   7x6x5           7x6
                         = 525 +            x6 +
                                   3x2x1           2x1
                         = (525 + 210 + 21)
                         = 756.

Answer:2 Option C

Explanation:
The word 'LEADING' has 7 different letters.

When the vowels EAI are always together, they can be supposed to
form one letter.

Then, we have to arrange the letters LNDG (EAI).

Now, 5 letters can be arranged in 5! = 120 ways.

The vowels (EAI) can be arranged among themselves in 3! = 6 ways.

  Required number of ways = (120 x 6) = 720.

Answer: 3 Option D

Explanation:

In the word 'CORPORATION', we treat the vowels OOAIO as one
letter.

Thus, we have CRPRTN (OOAIO).

This has 7 (6 + 1) letters of which R occurs 2 times and the rest are
different.

                                        7!
Number of ways arranging these letters = = 2520.
                                        2!

Now, 5 vowels in which O occurs 3 times and the rest are different,
can be arranged

  5!
in = 20 ways.
  3!

  Required number of ways = (2520 x 20) = 50400

Answer:4 Option C

Explanation:
Number of ways of selecting (3 consonants out of 7) and (2 vowels
out of 4)

   = (7C3 x 4C2)
      7x6x5 4x3
   =           x
      3x2x1 2x1
   = 210.

Number of groups, each having 3 consonants and 2 vowels = 210.

Each group contains 5 letters.

Number of ways of arranging
                            = 5!
5 letters among themselves
                            =5x4x3x2x1
                            = 120.

  Required number of ways = (210 x 120) = 25200.

Answer: 5Option C

Explanation:

The word 'LEADER' contains 6 letters, namely 1L, 2E, 1A, 1D and
1R.

  Required number of ways =6!=

Answer:6 Option D

Explanation:

We may have (1 boy and 3 girls) or (2 boys and 2 girls) or (3 boys
and 1 girl) or (4 boys).

  Required number 6
                 = ( C1 x 4C3) + (6C2 x 4C2) + (6C3 x 4C1) + (6C4)
of ways
                 = (6C1 x 4C1) + (6C2 x 4C2) + (6C3 x 4C1) + (6C2)
                           6x5 4x3         6x5x4            6x5
                 = (6 x 4)      x       +           x4 +
                           2x1 2x1         3x2x1            2x1
                 = (24 + 90 + 80 + 15)
= 209.

Answer:7 Option D

Explanation:

Since each desired number is divisible by 5, so we must have 5 at the
unit place. So, there is 1 way of doing it.

The tens place can now be filled by any of the remaining 5 digits (2,
3, 6, 7, 9). So, there are 5 ways of filling the tens place.

The hundreds place can now be filled by any of the remaining 4
digits. So, there are 4 ways of filling it.

           Required number of numbers = (1 x 5 x 4) = 20

Answer:8 Option C

           Explanation:

Required number of ways= (8C5 x 10C6)
                       = (8C3 x 10C4)
                           8 x 7 x 6 10 x 9 x 8 x 7
                       = 3 x 2 x 1x 4 x 3 x 2 x 1
                       = 11760.



Answer9: Option C

Explanation:

We may have(1 black and 2 non-black) or (2 black and 1 non-black)
or (3 black).

  Required number of ways= (3C1 x 6C2) + (3C2 x 6C1) + (3C3)
                               6x5      3x2
                         = 3x        +        x6 +1
                               2x1      2x1
                         = (45 + 18 + 1)
                         = 64.
Answer:10 Option C

Explanation:

There are 6 letters in the given word, out of which there are 3 vowels
and 3 consonants.

Let us mark these positions as under:

                         (1) (2) (3) (4) (5) (6)

Now, 3 vowels can be placed at any of the three places out 4, marked
1, 3, 5.

Number of ways of arranging the vowels = 3P3 = 3! = 6.

Also, the 3 consonants can be arranged at the remaining 3 positions.

Number of ways of these arrangements = 3P3 3! = 6.

Answer:11 Option A

Explanation:

                                                         7x
                               7     3        7    3
Required number of ways = ( C5 x C2) = ( C2 x C1)        6 x        =
                       =                                 2x 3      63.
                                                         1

Answer:12 Option C

Explanation:

'LOGARITHMS' contains 10 different letters.

Required number of = Number of arrangements of 10 letters, taking
words              4 at a time.
                   = 10P4
                   = (10 x 9 x 8 x 7)
                   = 5040.
Answer:13 Option C

Explanation:

In the word 'MATHEMATICS', we treat the vowels AEAI as one
letter.

Thus, we have MTHMTCS (AEAI).

Now, we have to arrange 8 letters, out of which M occurs twice, T
occurs twice and the rest are different.

                                                   8!
  Number of ways of arranging these letters =            = 10080.
                                                (2!)(2!)

Now, AEAI has 4 letters in which A occurs 2 times and the rest are
different.

                                           4!
Number of ways of arranging these letters = = 12.
                                           2!

  Required number of words = (10080 x 12) = 120960

Answer:14 Option B

Explanation:

The word 'OPTICAL' contains 7 different letters.

When the vowels OIA are always together, they can be supposed to
form one letter.

Then, we have to arrange the letters PTCL (OIA).

Now, 5 letters can be arranged in 5! = 120 ways.

The vowels (OIA) can be arranged among themselves in 3! = 6 ways.

  Required number of ways = (120 x 6) = 720.
ASSIGNMENT
 Question 1 The principal wants to arrange 5 students on the
platform such that the boy SALIM occupies the second position
and such that the girl SITA is always adjacent to the girl RITA .
How many such arrangements are possible?
Question 2 When a group- photograph is taken, all the seven
teachers should be in the first row and all the twenty students
should be in the second row. If the two corners of the second row
are reserved for the two tallest students, interchangeable only b/w
them, and if the middle seat of the front row is reserved for the
principal, how many such arrangements are possible?
Question 3 If there are six periods in each working day of a
school, in how many ways can one arrange 5 subjects such that
each subject is allotted at least one period?
Question 4 How many numbers greater than 4,00,000 can be
formed by using the digits 0, 2, 2, 4, 4, 5?
**Question 5 The letters of the word ZENITH are written in all
possible order. How many words are possible if all these words
are written out as in a dictionary? What is the rank of the word
ZENITH?
**Question 6 How many natural numbers not exceeding 4321 can
be formed with the digits 1,2,3 and 4 if the digits can repeat?
**Question 7 Three married couples are to be seated in a row
having six seats in a cinema hall. If spouses are to be seated next
to each other, in how many ways can they be seated? Find also the
number of ways of their seating if all the ladies sit together.
Question8 A boy has three library tickets and 8 books of his
interest in the library. Of these 8, he does not want to borrow
Chem. part II, unless Chem. Par I is also borrowed. In how many
ways can be choose the three books to be borrowed?
Question 9 A polygon has 44 diagonals. Find the number of its
sides.
** Question10 In how many ways can 10 things be equally divided
b/w (i) two persons (ii) two heaps ?
**Question11 In an exam. there are three multiple choice
questions and each question has 4 choices. Find the no. Of ways in
which a student fails to get all answer correct.
Question12 In how many ways can the letters of the word
PERMUTATIONS be arranged if the
(i) Words start with P and end with S? (ii) Vowels are all
together ? (iii) T’s are together?    (iv) There is no restriction?
  (v) There are always 4 letters b/w P & S?
Question 13 Find the no. Of perm. Of n things taken r at a time in
which 2 given things always occur. In how many pers. , they are
always excluded?
**Question 14 There are 10 points in a plane , no three of which
are in the same st. Line excepting 4 points, which are collinear.
Find the (i)
    No. Of st. Lines obtained from the pairs of these points , (ii)
no. Of ∆ that can be formed with the vertices as these points.
Question 15 Find the no. of per. Of n different things taken r at a
time such that two specific things occur together.

ANSWERS ( WITH HINTS)
Answer 1 since the boy SALIM occupies the second position, we
have to arrange the remaining 4 students, according to condition
two seats III, IV OR IV , V
         May be occupied by SITA & RITA in 4 ways, then
remaining seat may be occupied by 5th student in 1 way only.So
no. Of arrangements = 2.4.1=8.
Answer 2 The remaining 6 teachers can be arranged in the front
row in 6! Ways. (∵ middle seat is reserved for principal). The
remaining 18 students can be arranged in the second row in 18!
Ways. (two corner are reserved for 2 tallest , they can be occupied
2 ways) ∴ total no. Of ways = 6!.(18)!.2!
Answer 3 Five subjects can be allotted 5 periods out of the six
periods in 6P5 ways. Now one period is left and it can be allotted to
any one of the 5 subjects in 5 ways. So, total no. Of ways = 6P5 . 5
= 3600.
Answer 4 When 5 occurs the extreme left position(greater than 4) ,
then no. Of ways =       = 30. When 4 occurs at extreme left , then
no. Of ways = 5!/2!=60 ( 0,2,2,4,5[2 appears twice]) ∴ total no. Of
ways 90.
Answer 5 (i) total no. Of possible words = 6!=720
 (ii) The no. Of words beginning with E = 5! = 120 , similarly for
H ,I, N or T is 120.
      The words start with Z will also be 120 and will have their
rank from 601 to 720.Of these 120 words start with Z, the no. Of
words with E in the second place is 120/5 = 24. ZE**** will have
their rank from 601 to 624. Of these 24 words , NO. Of words
with H(or I or N or T) in the third place is 24/2=6
   Rank order : ZEH*** , ZEI***, ZEN*** 601 to 606, 607 to
612, 613 to 618 resp., start with ZEN are 613...ZENHIT, 614...
ZENHTI, 615... ZENIHT,
  616... ZENITH.
Answer 6 1- digit nos. = 4, 2-digit nos. = 4x16, 3-digit nos. = 64,
4-digit nos. 256. Out of 256 , let us discard those which are greater
than 4321.
    (i) 4 at thousand’s place & hundred’s place= 1.1.4.4=16 ,
               4 4 1 1
                      2 2
                      3 3
                      4 4


Similarly (ii) 4 at thousand’s place ,3 at hundred’s place, 3 or 4 in
ten’s = 1.1.2.4=8,
 (iii) 4 at thousand’s place ,3 at hundred’s place, 2 in ten’s & 2,3,4
at unit’s= 1.1.1.3=3. Total 4-digit no.greater than 4321 are = 27,
answer is 256-27=313.
Answer 7(i) Let A, B, C be three married couples can be seated =
3! & can sit in 2 ways =2! ∴ req. Ways of seating = 3!.2!.2!.2!=48 .
     (ii) three ladies can be seated in 4 ways which are at seat
number (1,2,3), (2,3,4), (3,4,5), (4,5,6). They can interchange their
seats =3! Ways. Men can be seated at three remaining seats in 3! ,
so req. No. Of ways = 4.3!.3!= 144 .
Answer 8 case-1 he borrows chem.. part II, chem.. part I and one
more book out of the remaining 6 (8-2) books.
Case-2 he does not borrow chem.. part-II and , so
borrow all 3 books out of 7 ∴ total no. Of ways = 6C1 + 7C3=41.
Answer 9 No. Of diagonals = nC2 - n = 44 ( nC2 = no.of st. Lines
of polygon n sides).
Answer 10 (i) 10 things be equally divided b/w two persons,
groups are distinct , 10!/ (5!)2 = 252 (ii) 10 things be equally
divided b/w two heaps no distinction can be made = ½ (252)=126.
 Answer 11 Req. No. Of ways= 4.4.4 – 1=63.
Answer 13 (i) 10!/2! (ii) consider 5 vowels as one letter (8!/2!)
.(5!) (iii) consider 2 T’s as one letter = (11)! (iv) 12!/2!
    (v) 12 letters in 12 places as P****S****** leaving 4 places in
between , Thus P & S may be filled up in 7 ways, same for S & P ,
remaining 10 letters are arranged in 10!/2! Ways ∴ req. No. of
ways 10!/(3!.2!.2!) = 151200.
 Answer 14 No. Of per. Of n things taken r at a time is same as the
no. Of ways of filling up r places with n things, we arrange 2
things which can be arranged in r places in rP2 ways. Also
remaining (r-2) places can be filled up with remaining (n-2) things
in n-2Pr-2 ways.
No. Of per. (2 are included) = rp2 . n-2Pr-2 , No. Of per. (2 are not
included) = n-2Pr
Answer 15 (i) No. Of st. Lines formed by joining 10 points ,
taking 2 at a time = 10C2 = 45. No. Of st. Lines joining 4 points =
4
  C2 = 6 , but 4 are collinear , when join pairwise give only one line.
Req. No. of lines = 45 – 6 + 1=40
  (ii) No. of ∆s formed by joining the points , taken 3 at a time =
10
   C3 = 120, no. of ∆s formed by joining 4 points , taken 3 at a time
= 4C3 = 4
    Req. No. of ∆s = 120 – 4 = 116.
Answer 16 Two specific things can be in r places in (r-1) ways and
two things can be arranged among themselves in 2! Ways and the
remaining (n-2) things will be arranged in (r-2) places in n-2Pr-2
ways. Req. Per. = 2! (r-1) . n-2Pr-2 .
Combinations and permutations

Más contenido relacionado

La actualidad más candente

Basic Concept Of Probability
Basic Concept Of ProbabilityBasic Concept Of Probability
Basic Concept Of Probabilityguest45a926
 
2.3 Distance and Midpoint Formulas
2.3 Distance and Midpoint Formulas2.3 Distance and Midpoint Formulas
2.3 Distance and Midpoint Formulassmiller5
 
Basic concept of probability
Basic concept of probabilityBasic concept of probability
Basic concept of probabilityIkhlas Rahman
 
permutations power point
permutations power pointpermutations power point
permutations power pointAldrin Balenton
 
Measures of position. gouped data
Measures of position. gouped dataMeasures of position. gouped data
Measures of position. gouped dataArmathia Sambrano
 
Right triangles
Right trianglesRight triangles
Right triangleszanstett
 
Counting techniques
Counting techniquesCounting techniques
Counting techniquesChie Pegollo
 
The mobius function and the mobius inversion formula
The mobius function and the mobius inversion formulaThe mobius function and the mobius inversion formula
The mobius function and the mobius inversion formulaBenjJamiesonDuag
 
Midline Theorem Math 9.pptx
Midline Theorem Math 9.pptxMidline Theorem Math 9.pptx
Midline Theorem Math 9.pptxerickcabutaje1
 
Permutations and Combinations (All Formulas)
Permutations and Combinations (All Formulas)Permutations and Combinations (All Formulas)
Permutations and Combinations (All Formulas)Anubhav Kumar
 
11.3 Combinations
11.3 Combinations11.3 Combinations
11.3 CombinationsRyan Pineda
 
Arc length, area of a sector and segments of a circle
Arc length, area of a sector and segments of a circleArc length, area of a sector and segments of a circle
Arc length, area of a sector and segments of a circleJoey Valdriz
 
Permutation & Combination
Permutation & CombinationPermutation & Combination
Permutation & CombinationPuru Agrawal
 
Combinations and permutations(1)
Combinations and permutations(1)Combinations and permutations(1)
Combinations and permutations(1)Abebaw Abun Amanu
 
11.1 Fundamental Counting Principle
11.1 Fundamental Counting Principle11.1 Fundamental Counting Principle
11.1 Fundamental Counting PrincipleRyan Pineda
 
Lesson 8 conic sections - parabola
Lesson 8    conic sections - parabolaLesson 8    conic sections - parabola
Lesson 8 conic sections - parabolaJean Leano
 

La actualidad más candente (20)

Basic Concept Of Probability
Basic Concept Of ProbabilityBasic Concept Of Probability
Basic Concept Of Probability
 
2.3 Distance and Midpoint Formulas
2.3 Distance and Midpoint Formulas2.3 Distance and Midpoint Formulas
2.3 Distance and Midpoint Formulas
 
Basic concept of probability
Basic concept of probabilityBasic concept of probability
Basic concept of probability
 
permutations power point
permutations power pointpermutations power point
permutations power point
 
Measures of position. gouped data
Measures of position. gouped dataMeasures of position. gouped data
Measures of position. gouped data
 
Right triangles
Right trianglesRight triangles
Right triangles
 
Counting techniques
Counting techniquesCounting techniques
Counting techniques
 
Mathematical Logic
Mathematical LogicMathematical Logic
Mathematical Logic
 
The mobius function and the mobius inversion formula
The mobius function and the mobius inversion formulaThe mobius function and the mobius inversion formula
The mobius function and the mobius inversion formula
 
Midline Theorem Math 9.pptx
Midline Theorem Math 9.pptxMidline Theorem Math 9.pptx
Midline Theorem Math 9.pptx
 
Permutations and Combinations (All Formulas)
Permutations and Combinations (All Formulas)Permutations and Combinations (All Formulas)
Permutations and Combinations (All Formulas)
 
11.3 Combinations
11.3 Combinations11.3 Combinations
11.3 Combinations
 
Arc length, area of a sector and segments of a circle
Arc length, area of a sector and segments of a circleArc length, area of a sector and segments of a circle
Arc length, area of a sector and segments of a circle
 
Permutation & Combination
Permutation & CombinationPermutation & Combination
Permutation & Combination
 
Combinations and permutations(1)
Combinations and permutations(1)Combinations and permutations(1)
Combinations and permutations(1)
 
Binomial probability distributions
Binomial probability distributions  Binomial probability distributions
Binomial probability distributions
 
11.1 Fundamental Counting Principle
11.1 Fundamental Counting Principle11.1 Fundamental Counting Principle
11.1 Fundamental Counting Principle
 
Lesson 8 conic sections - parabola
Lesson 8    conic sections - parabolaLesson 8    conic sections - parabola
Lesson 8 conic sections - parabola
 
1541 infinite limits
1541 infinite limits1541 infinite limits
1541 infinite limits
 
Unit 2: All
Unit 2: AllUnit 2: All
Unit 2: All
 

Destacado

Quantitative Aptitude And Mathematics 14 October Ii
Quantitative Aptitude And Mathematics 14 October IiQuantitative Aptitude And Mathematics 14 October Ii
Quantitative Aptitude And Mathematics 14 October IiDr. Trilok Kumar Jain
 
30. premutation & combinations
30. premutation & combinations30. premutation & combinations
30. premutation & combinationsAkhilesh Sharma
 
Discrete maths questions
Discrete maths questionsDiscrete maths questions
Discrete maths questionsDIT University
 
Call Center SESI
Call Center SESICall Center SESI
Call Center SESIMarco Coghi
 
Alexandr vannas sof Facilities Management, Virtual Reality and the Internet o...
Alexandr vannas sof Facilities Management, Virtual Reality and the Internet o...Alexandr vannas sof Facilities Management, Virtual Reality and the Internet o...
Alexandr vannas sof Facilities Management, Virtual Reality and the Internet o...Business of Software Conference
 
12 Esempi di Contenuti di Valore "Pesanti"
12 Esempi di Contenuti di Valore "Pesanti"12 Esempi di Contenuti di Valore "Pesanti"
12 Esempi di Contenuti di Valore "Pesanti"Robin Good
 
Informe radiográfico de radiografía panorámica
Informe radiográfico de radiografía panorámicaInforme radiográfico de radiografía panorámica
Informe radiográfico de radiografía panorámicaKelly Andia
 
Chapter 3 Breaking up long sentences (ENGLISH FOR WRITING RESEARCH PAPERS)
Chapter 3 Breaking up long sentences (ENGLISH FOR WRITING RESEARCH PAPERS)Chapter 3 Breaking up long sentences (ENGLISH FOR WRITING RESEARCH PAPERS)
Chapter 3 Breaking up long sentences (ENGLISH FOR WRITING RESEARCH PAPERS)Hafiza Abas
 
Arens14e ch01 ppt
Arens14e ch01 pptArens14e ch01 ppt
Arens14e ch01 pptsrkndnz
 
Changing word order in sentences
Changing word order in sentences Changing word order in sentences
Changing word order in sentences cpitois
 
Outcome of World War I
Outcome of World War IOutcome of World War I
Outcome of World War Illfrye11
 
D p ii fixação do regime inicial de cumprimento da pena privativa de liberdade
D p  ii  fixação do regime inicial de cumprimento da pena privativa de liberdadeD p  ii  fixação do regime inicial de cumprimento da pena privativa de liberdade
D p ii fixação do regime inicial de cumprimento da pena privativa de liberdadeJames Martins Pereira
 
Pratt 5 Teaching Perspectives
Pratt   5 Teaching PerspectivesPratt   5 Teaching Perspectives
Pratt 5 Teaching Perspectivesmharttrup
 
Ideology & Science
Ideology & ScienceIdeology & Science
Ideology & ScienceBeth Lee
 
Image is Everything: ipadpalooza16
Image is Everything: ipadpalooza16Image is Everything: ipadpalooza16
Image is Everything: ipadpalooza16Amy Burvall
 
Operators’ Positioning as a Digital Lifestyle Solution Provider
Operators’ Positioning as a Digital Lifestyle Solution ProviderOperators’ Positioning as a Digital Lifestyle Solution Provider
Operators’ Positioning as a Digital Lifestyle Solution ProviderAli Saghaeian
 

Destacado (19)

Quantitative Aptitude And Mathematics 14 October Ii
Quantitative Aptitude And Mathematics 14 October IiQuantitative Aptitude And Mathematics 14 October Ii
Quantitative Aptitude And Mathematics 14 October Ii
 
30. premutation & combinations
30. premutation & combinations30. premutation & combinations
30. premutation & combinations
 
Discrete maths questions
Discrete maths questionsDiscrete maths questions
Discrete maths questions
 
Wenn, ach wenn
Wenn, ach wennWenn, ach wenn
Wenn, ach wenn
 
Call Center SESI
Call Center SESICall Center SESI
Call Center SESI
 
Do you have english class today
Do you have english class todayDo you have english class today
Do you have english class today
 
Alexandr vannas sof Facilities Management, Virtual Reality and the Internet o...
Alexandr vannas sof Facilities Management, Virtual Reality and the Internet o...Alexandr vannas sof Facilities Management, Virtual Reality and the Internet o...
Alexandr vannas sof Facilities Management, Virtual Reality and the Internet o...
 
12 Esempi di Contenuti di Valore "Pesanti"
12 Esempi di Contenuti di Valore "Pesanti"12 Esempi di Contenuti di Valore "Pesanti"
12 Esempi di Contenuti di Valore "Pesanti"
 
Informe radiográfico de radiografía panorámica
Informe radiográfico de radiografía panorámicaInforme radiográfico de radiografía panorámica
Informe radiográfico de radiografía panorámica
 
Chapter 3 Breaking up long sentences (ENGLISH FOR WRITING RESEARCH PAPERS)
Chapter 3 Breaking up long sentences (ENGLISH FOR WRITING RESEARCH PAPERS)Chapter 3 Breaking up long sentences (ENGLISH FOR WRITING RESEARCH PAPERS)
Chapter 3 Breaking up long sentences (ENGLISH FOR WRITING RESEARCH PAPERS)
 
Arens14e ch01 ppt
Arens14e ch01 pptArens14e ch01 ppt
Arens14e ch01 ppt
 
Changing word order in sentences
Changing word order in sentences Changing word order in sentences
Changing word order in sentences
 
Outcome of World War I
Outcome of World War IOutcome of World War I
Outcome of World War I
 
D p ii fixação do regime inicial de cumprimento da pena privativa de liberdade
D p  ii  fixação do regime inicial de cumprimento da pena privativa de liberdadeD p  ii  fixação do regime inicial de cumprimento da pena privativa de liberdade
D p ii fixação do regime inicial de cumprimento da pena privativa de liberdade
 
Pratt 5 Teaching Perspectives
Pratt   5 Teaching PerspectivesPratt   5 Teaching Perspectives
Pratt 5 Teaching Perspectives
 
Ideology & Science
Ideology & ScienceIdeology & Science
Ideology & Science
 
Champions of design vol #1
Champions of design vol #1Champions of design vol #1
Champions of design vol #1
 
Image is Everything: ipadpalooza16
Image is Everything: ipadpalooza16Image is Everything: ipadpalooza16
Image is Everything: ipadpalooza16
 
Operators’ Positioning as a Digital Lifestyle Solution Provider
Operators’ Positioning as a Digital Lifestyle Solution ProviderOperators’ Positioning as a Digital Lifestyle Solution Provider
Operators’ Positioning as a Digital Lifestyle Solution Provider
 

Similar a Combinations and permutations

Counting technique
Counting techniqueCounting technique
Counting techniqueNadeem Uddin
 
Chapter-3-Sample-Space-of-Experiment.pdf
Chapter-3-Sample-Space-of-Experiment.pdfChapter-3-Sample-Space-of-Experiment.pdf
Chapter-3-Sample-Space-of-Experiment.pdfJuliusBoitizon
 
Permutations and-combinations-maths
Permutations and-combinations-mathsPermutations and-combinations-maths
Permutations and-combinations-mathsMurugan Iron
 
Lecture Week 17 which hleps in study for logic and
Lecture Week 17 which hleps in study for logic andLecture Week 17 which hleps in study for logic and
Lecture Week 17 which hleps in study for logic andmanishhmishra001
 
Permutation and combinations
Permutation and combinationsPermutation and combinations
Permutation and combinationsRushabh Vora
 
Permutation & combination
Permutation & combinationPermutation & combination
Permutation & combinationAPEX INSTITUTE
 
Introduction to probability solutions manual
Introduction to probability   solutions manualIntroduction to probability   solutions manual
Introduction to probability solutions manualKibria Prangon
 
Arithmetic sequences and series[1]
Arithmetic sequences and series[1]Arithmetic sequences and series[1]
Arithmetic sequences and series[1]indu psthakur
 
Permutations and Combinations(For Class 11)
Permutations and Combinations(For Class 11)Permutations and Combinations(For Class 11)
Permutations and Combinations(For Class 11)Anubhav Kumar
 
1. Let D be the random variable with value equal to the number showi.pdf
1. Let D be the random variable with value equal to the number showi.pdf1. Let D be the random variable with value equal to the number showi.pdf
1. Let D be the random variable with value equal to the number showi.pdfakramali786
 
Week 2: Arithmetic sequence
Week 2:  Arithmetic sequenceWeek 2:  Arithmetic sequence
Week 2: Arithmetic sequenceRozzel Palacio
 
Permutations and Combinations IIT JEE+Olympiad Lecture 3
Permutations and Combinations IIT JEE+Olympiad Lecture 3 Permutations and Combinations IIT JEE+Olympiad Lecture 3
Permutations and Combinations IIT JEE+Olympiad Lecture 3 Parth Nandedkar
 

Similar a Combinations and permutations (20)

Counting technique
Counting techniqueCounting technique
Counting technique
 
Chapter-3-Sample-Space-of-Experiment.pdf
Chapter-3-Sample-Space-of-Experiment.pdfChapter-3-Sample-Space-of-Experiment.pdf
Chapter-3-Sample-Space-of-Experiment.pdf
 
Permutations and-combinations-maths
Permutations and-combinations-mathsPermutations and-combinations-maths
Permutations and-combinations-maths
 
Lecture Week 17 which hleps in study for logic and
Lecture Week 17 which hleps in study for logic andLecture Week 17 which hleps in study for logic and
Lecture Week 17 which hleps in study for logic and
 
Day 3 examples
Day 3 examplesDay 3 examples
Day 3 examples
 
New day 3 examples
New day 3 examplesNew day 3 examples
New day 3 examples
 
Day 3 examples
Day 3 examplesDay 3 examples
Day 3 examples
 
Permutation and combinations
Permutation and combinationsPermutation and combinations
Permutation and combinations
 
Permutation & combination
Permutation & combinationPermutation & combination
Permutation & combination
 
Permutations
PermutationsPermutations
Permutations
 
Introduction to probability solutions manual
Introduction to probability   solutions manualIntroduction to probability   solutions manual
Introduction to probability solutions manual
 
Bba ii-u1-p&c
Bba ii-u1-p&cBba ii-u1-p&c
Bba ii-u1-p&c
 
Arithmetic sequences and series[1]
Arithmetic sequences and series[1]Arithmetic sequences and series[1]
Arithmetic sequences and series[1]
 
Ch7
Ch7Ch7
Ch7
 
Permutations and Combinations(For Class 11)
Permutations and Combinations(For Class 11)Permutations and Combinations(For Class 11)
Permutations and Combinations(For Class 11)
 
Permutation
PermutationPermutation
Permutation
 
1. Let D be the random variable with value equal to the number showi.pdf
1. Let D be the random variable with value equal to the number showi.pdf1. Let D be the random variable with value equal to the number showi.pdf
1. Let D be the random variable with value equal to the number showi.pdf
 
Week 2: Arithmetic sequence
Week 2:  Arithmetic sequenceWeek 2:  Arithmetic sequence
Week 2: Arithmetic sequence
 
Ch08
Ch08Ch08
Ch08
 
Permutations and Combinations IIT JEE+Olympiad Lecture 3
Permutations and Combinations IIT JEE+Olympiad Lecture 3 Permutations and Combinations IIT JEE+Olympiad Lecture 3
Permutations and Combinations IIT JEE+Olympiad Lecture 3
 

Más de indu psthakur

Class xii practice questions
Class xii practice questionsClass xii practice questions
Class xii practice questionsindu psthakur
 
Practice questions( calculus ) xii
Practice questions( calculus ) xiiPractice questions( calculus ) xii
Practice questions( calculus ) xiiindu psthakur
 
Question bank -xi (hots)
Question bank -xi (hots)Question bank -xi (hots)
Question bank -xi (hots)indu psthakur
 
Linear ineqns. and statistics
Linear ineqns. and statisticsLinear ineqns. and statistics
Linear ineqns. and statisticsindu psthakur
 
Relations & functions.pps
Relations  &  functions.ppsRelations  &  functions.pps
Relations & functions.ppsindu psthakur
 
Binomial theorem for any index
Binomial theorem for any indexBinomial theorem for any index
Binomial theorem for any indexindu psthakur
 
Series expansion of exponential and logarithmic functions
Series expansion of exponential and logarithmic functionsSeries expansion of exponential and logarithmic functions
Series expansion of exponential and logarithmic functionsindu psthakur
 
Solution of triangles
Solution of trianglesSolution of triangles
Solution of trianglesindu psthakur
 
Trigonometry for class xi
Trigonometry for class xiTrigonometry for class xi
Trigonometry for class xiindu psthakur
 
Mathmatical reasoning
Mathmatical reasoningMathmatical reasoning
Mathmatical reasoningindu psthakur
 
Assessments for class xi
Assessments  for class  xi Assessments  for class  xi
Assessments for class xi indu psthakur
 

Más de indu psthakur (18)

Class xii practice questions
Class xii practice questionsClass xii practice questions
Class xii practice questions
 
Practice questions( calculus ) xii
Practice questions( calculus ) xiiPractice questions( calculus ) xii
Practice questions( calculus ) xii
 
Question bank -xi (hots)
Question bank -xi (hots)Question bank -xi (hots)
Question bank -xi (hots)
 
Question bank xi
Question bank xiQuestion bank xi
Question bank xi
 
Assignmen ts --x
Assignmen ts  --xAssignmen ts  --x
Assignmen ts --x
 
Binomial theorem
Binomial theoremBinomial theorem
Binomial theorem
 
Linear ineqns. and statistics
Linear ineqns. and statisticsLinear ineqns. and statistics
Linear ineqns. and statistics
 
Conic sections
Conic sectionsConic sections
Conic sections
 
Straight lines
Straight linesStraight lines
Straight lines
 
Relations & functions.pps
Relations  &  functions.ppsRelations  &  functions.pps
Relations & functions.pps
 
Binomial theorem for any index
Binomial theorem for any indexBinomial theorem for any index
Binomial theorem for any index
 
Series expansion of exponential and logarithmic functions
Series expansion of exponential and logarithmic functionsSeries expansion of exponential and logarithmic functions
Series expansion of exponential and logarithmic functions
 
Solution of triangles
Solution of trianglesSolution of triangles
Solution of triangles
 
Trigonometry for class xi
Trigonometry for class xiTrigonometry for class xi
Trigonometry for class xi
 
Mathmatical reasoning
Mathmatical reasoningMathmatical reasoning
Mathmatical reasoning
 
Sets
SetsSets
Sets
 
Complex numbers
Complex numbersComplex numbers
Complex numbers
 
Assessments for class xi
Assessments  for class  xi Assessments  for class  xi
Assessments for class xi
 

Último

Beyond the EU: DORA and NIS 2 Directive's Global Impact
Beyond the EU: DORA and NIS 2 Directive's Global ImpactBeyond the EU: DORA and NIS 2 Directive's Global Impact
Beyond the EU: DORA and NIS 2 Directive's Global ImpactPECB
 
Paris 2024 Olympic Geographies - an activity
Paris 2024 Olympic Geographies - an activityParis 2024 Olympic Geographies - an activity
Paris 2024 Olympic Geographies - an activityGeoBlogs
 
mini mental status format.docx
mini    mental       status     format.docxmini    mental       status     format.docx
mini mental status format.docxPoojaSen20
 
Student login on Anyboli platform.helpin
Student login on Anyboli platform.helpinStudent login on Anyboli platform.helpin
Student login on Anyboli platform.helpinRaunakKeshri1
 
Q4-W6-Restating Informational Text Grade 3
Q4-W6-Restating Informational Text Grade 3Q4-W6-Restating Informational Text Grade 3
Q4-W6-Restating Informational Text Grade 3JemimahLaneBuaron
 
Software Engineering Methodologies (overview)
Software Engineering Methodologies (overview)Software Engineering Methodologies (overview)
Software Engineering Methodologies (overview)eniolaolutunde
 
Nutritional Needs Presentation - HLTH 104
Nutritional Needs Presentation - HLTH 104Nutritional Needs Presentation - HLTH 104
Nutritional Needs Presentation - HLTH 104misteraugie
 
Presentation by Andreas Schleicher Tackling the School Absenteeism Crisis 30 ...
Presentation by Andreas Schleicher Tackling the School Absenteeism Crisis 30 ...Presentation by Andreas Schleicher Tackling the School Absenteeism Crisis 30 ...
Presentation by Andreas Schleicher Tackling the School Absenteeism Crisis 30 ...EduSkills OECD
 
Activity 01 - Artificial Culture (1).pdf
Activity 01 - Artificial Culture (1).pdfActivity 01 - Artificial Culture (1).pdf
Activity 01 - Artificial Culture (1).pdfciinovamais
 
Grant Readiness 101 TechSoup and Remy Consulting
Grant Readiness 101 TechSoup and Remy ConsultingGrant Readiness 101 TechSoup and Remy Consulting
Grant Readiness 101 TechSoup and Remy ConsultingTechSoup
 
Introduction to Nonprofit Accounting: The Basics
Introduction to Nonprofit Accounting: The BasicsIntroduction to Nonprofit Accounting: The Basics
Introduction to Nonprofit Accounting: The BasicsTechSoup
 
Sports & Fitness Value Added Course FY..
Sports & Fitness Value Added Course FY..Sports & Fitness Value Added Course FY..
Sports & Fitness Value Added Course FY..Disha Kariya
 
A Critique of the Proposed National Education Policy Reform
A Critique of the Proposed National Education Policy ReformA Critique of the Proposed National Education Policy Reform
A Critique of the Proposed National Education Policy ReformChameera Dedduwage
 
Kisan Call Centre - To harness potential of ICT in Agriculture by answer farm...
Kisan Call Centre - To harness potential of ICT in Agriculture by answer farm...Kisan Call Centre - To harness potential of ICT in Agriculture by answer farm...
Kisan Call Centre - To harness potential of ICT in Agriculture by answer farm...Krashi Coaching
 
BASLIQ CURRENT LOOKBOOK LOOKBOOK(1) (1).pdf
BASLIQ CURRENT LOOKBOOK  LOOKBOOK(1) (1).pdfBASLIQ CURRENT LOOKBOOK  LOOKBOOK(1) (1).pdf
BASLIQ CURRENT LOOKBOOK LOOKBOOK(1) (1).pdfSoniaTolstoy
 
Measures of Dispersion and Variability: Range, QD, AD and SD
Measures of Dispersion and Variability: Range, QD, AD and SDMeasures of Dispersion and Variability: Range, QD, AD and SD
Measures of Dispersion and Variability: Range, QD, AD and SDThiyagu K
 
Disha NEET Physics Guide for classes 11 and 12.pdf
Disha NEET Physics Guide for classes 11 and 12.pdfDisha NEET Physics Guide for classes 11 and 12.pdf
Disha NEET Physics Guide for classes 11 and 12.pdfchloefrazer622
 
Sanyam Choudhary Chemistry practical.pdf
Sanyam Choudhary Chemistry practical.pdfSanyam Choudhary Chemistry practical.pdf
Sanyam Choudhary Chemistry practical.pdfsanyamsingh5019
 
CARE OF CHILD IN INCUBATOR..........pptx
CARE OF CHILD IN INCUBATOR..........pptxCARE OF CHILD IN INCUBATOR..........pptx
CARE OF CHILD IN INCUBATOR..........pptxGaneshChakor2
 

Último (20)

Beyond the EU: DORA and NIS 2 Directive's Global Impact
Beyond the EU: DORA and NIS 2 Directive's Global ImpactBeyond the EU: DORA and NIS 2 Directive's Global Impact
Beyond the EU: DORA and NIS 2 Directive's Global Impact
 
Paris 2024 Olympic Geographies - an activity
Paris 2024 Olympic Geographies - an activityParis 2024 Olympic Geographies - an activity
Paris 2024 Olympic Geographies - an activity
 
mini mental status format.docx
mini    mental       status     format.docxmini    mental       status     format.docx
mini mental status format.docx
 
Código Creativo y Arte de Software | Unidad 1
Código Creativo y Arte de Software | Unidad 1Código Creativo y Arte de Software | Unidad 1
Código Creativo y Arte de Software | Unidad 1
 
Student login on Anyboli platform.helpin
Student login on Anyboli platform.helpinStudent login on Anyboli platform.helpin
Student login on Anyboli platform.helpin
 
Q4-W6-Restating Informational Text Grade 3
Q4-W6-Restating Informational Text Grade 3Q4-W6-Restating Informational Text Grade 3
Q4-W6-Restating Informational Text Grade 3
 
Software Engineering Methodologies (overview)
Software Engineering Methodologies (overview)Software Engineering Methodologies (overview)
Software Engineering Methodologies (overview)
 
Nutritional Needs Presentation - HLTH 104
Nutritional Needs Presentation - HLTH 104Nutritional Needs Presentation - HLTH 104
Nutritional Needs Presentation - HLTH 104
 
Presentation by Andreas Schleicher Tackling the School Absenteeism Crisis 30 ...
Presentation by Andreas Schleicher Tackling the School Absenteeism Crisis 30 ...Presentation by Andreas Schleicher Tackling the School Absenteeism Crisis 30 ...
Presentation by Andreas Schleicher Tackling the School Absenteeism Crisis 30 ...
 
Activity 01 - Artificial Culture (1).pdf
Activity 01 - Artificial Culture (1).pdfActivity 01 - Artificial Culture (1).pdf
Activity 01 - Artificial Culture (1).pdf
 
Grant Readiness 101 TechSoup and Remy Consulting
Grant Readiness 101 TechSoup and Remy ConsultingGrant Readiness 101 TechSoup and Remy Consulting
Grant Readiness 101 TechSoup and Remy Consulting
 
Introduction to Nonprofit Accounting: The Basics
Introduction to Nonprofit Accounting: The BasicsIntroduction to Nonprofit Accounting: The Basics
Introduction to Nonprofit Accounting: The Basics
 
Sports & Fitness Value Added Course FY..
Sports & Fitness Value Added Course FY..Sports & Fitness Value Added Course FY..
Sports & Fitness Value Added Course FY..
 
A Critique of the Proposed National Education Policy Reform
A Critique of the Proposed National Education Policy ReformA Critique of the Proposed National Education Policy Reform
A Critique of the Proposed National Education Policy Reform
 
Kisan Call Centre - To harness potential of ICT in Agriculture by answer farm...
Kisan Call Centre - To harness potential of ICT in Agriculture by answer farm...Kisan Call Centre - To harness potential of ICT in Agriculture by answer farm...
Kisan Call Centre - To harness potential of ICT in Agriculture by answer farm...
 
BASLIQ CURRENT LOOKBOOK LOOKBOOK(1) (1).pdf
BASLIQ CURRENT LOOKBOOK  LOOKBOOK(1) (1).pdfBASLIQ CURRENT LOOKBOOK  LOOKBOOK(1) (1).pdf
BASLIQ CURRENT LOOKBOOK LOOKBOOK(1) (1).pdf
 
Measures of Dispersion and Variability: Range, QD, AD and SD
Measures of Dispersion and Variability: Range, QD, AD and SDMeasures of Dispersion and Variability: Range, QD, AD and SD
Measures of Dispersion and Variability: Range, QD, AD and SD
 
Disha NEET Physics Guide for classes 11 and 12.pdf
Disha NEET Physics Guide for classes 11 and 12.pdfDisha NEET Physics Guide for classes 11 and 12.pdf
Disha NEET Physics Guide for classes 11 and 12.pdf
 
Sanyam Choudhary Chemistry practical.pdf
Sanyam Choudhary Chemistry practical.pdfSanyam Choudhary Chemistry practical.pdf
Sanyam Choudhary Chemistry practical.pdf
 
CARE OF CHILD IN INCUBATOR..........pptx
CARE OF CHILD IN INCUBATOR..........pptxCARE OF CHILD IN INCUBATOR..........pptx
CARE OF CHILD IN INCUBATOR..........pptx
 

Combinations and permutations

  • 1. Combinations and Permutations What's the Difference? In English we use the word "combination" loosely, without thinking if the order of things is important. In other words: "My fruit salad is a combination of apples, grapes and bananas" We don't care what order the fruits are in, they could also be "bananas, grapes and apples" or "grapes, apples and bananas", its the same fruit salad. "The combination to the safe was 472". Now we do care about the order. "724" would not work, nor would "247". It has to be exactly 4-7-2. So, in Mathematics we use more precise language: If the order doesn't matter, it is a Combination. If the order does matter it is a Permutation. A Permutation is an ordered Combination. Permutation : Permutation means arrangement of things. The word arrangement is used, if the order of things is considered. Combination: Combination means selection of things. The word selection is used, when the order of things has no importance. Definition: Permutation: An arrangement is called a Permutation. It is the rearrangement of objects or symbols into distinguishable sequences. When we set things in order, we say we have made an arrangement. When we change the order, we say we have changed the
  • 2. arrangement. So each of the arrangement that can be made by taking some or all of a number of things is known as Permutation. Combination: A Combination is a selection of some or all of a number of different objects. It is an un-ordered collection of unique sizes.In a permutation the order of occurence of the objects or the arrangement is important but in combination the order of occurence of the objects is not important. Formula: Permutation = nPr = n! / (n-r)! Combination = nCr = nPr / r! where, n, r are non negative integers and 0≤ r≤n. r is the size of each permutation. n is the size of the set from which elements are permuted. ! is the factorial operator. Example: Suppose we have to form a number of consisting of three digits using the digits 1,2,3,4, To form this number the digits have to be arranged. Different numbers will get formed depending upon the order in which we arrange the digits. This is an example of Permutation. Now suppose that we have to make a team of 11 players out of 20 players, This is an example of combination, because the order of players in the team will not result in a change in the team. No matter in which order we list out the players the team will remain the same! For a different team to be formed at least one player will have to be changed. Now let us look at two fundamental principles of counting:
  • 3. Addition rule : If an experiment can be performed in „n‟ ways, & another experiment can be performed in „m‟ ways then either of the two experiments can be performed in (m+n) ways. This rule can be extended to any finite number of experiments. Example: Suppose there are 3 doors in a room, 2 on one side and 1 on other side. A man wants to go out from the room. Obviously he has „3‟ options for it. He can come out by door „A‟ or door „B‟ or door ‟C‟. Multiplication Rule : If a work can be done in m ways, another work can be done in „n‟ ways, then both of the operations can be performed in m x n ways. It can be extended to any finite number of operations. Example.: Suppose a man wants to cross-out a room, which has 2 doors on one side and 1 door on other site. He has 2 x 1 = 2 ways for it. Factorial n : The product of first „n‟ natural numbers is denoted by n!. n! = n(n-1) (n-2) ………………..3.2.1. Ex. 5! = 5 x 4 x 3 x 2 x 1 =120 Note 0! = 1
  • 4. Proof n! =n, (n-1)! Or (n-1)! = [n x (n-1)!]/n = n! /n Putting n = 1, we have O! = 1!/1 = 1 Permutation Number of permutations of „n‟ different things taken „r‟ at a time is given by:- n Pr = n!/(n-r)! Proof: Say we have „n‟ different things a1, a2……, an. Clearly the first place can be filled up in „n‟ ways. Number of things left after filling-up the first place = n-1 So the second-place can be filled-up in (n-1) ways. Now number of things left after filling-up the first and second places = n - 2 Now the third place can be filled-up in (n-2) ways. Thus number of ways of filling-up first-place = n Number of ways of filling-up second-place = n-1 Number of ways of filling-up third-place = n-2 Number of ways of filling-up r-th place = n – (r-1) = n-r+1 By multiplication – rule of counting, total no. of ways of filling up, first, second -- rth-place together :-
  • 5. n (n-1) (n-2) ------------ (n-r+1) Hence: n Pr = n (n-1)(n-2) --------------(n-r+1) = [n(n-1)(n-2)----------(n-r+1)] [(n-r)(n-r-1)-----3.2.1.] / [(n-r)(n-r- 1)] ----3.2.1 n Pr = n!/(n-r)! Number of permutations of „n‟ different things taken all at a time is given by:- n Pn = n! Proof : Now we have „n‟ objects, and n-places. Number of ways of filling-up first-place = n Number of ways of filling-up second-place = n-1 Number of ways of filling-up third-place = n-2 Number of ways of filling-up r-th place, i.e. last place =1 Number of ways of filling-up first, second, --- n th place = n (n-1) (n-2) ------ 2.1. n Pn = n! Concept. n We have Pr = n!/n-r Putting r = n, we have :-
  • 6. n Pr = n! / (n-r) n But Pn = n! Clearly it is possible, only when n! = 1 Hence it is proof that 0! = 1 Note : Factorial of negative-number is not defined. The expression –3! has no meaning. Examples Q. How many different signals can be made by 5 flags from 8- flags of different colours? Ans. Number of ways taking 5 flags out of 8-flage = 8P5 = 8!/(8-5)! = 8 x 7 x 6 x 5 x 4 = 6720 Q. How many words can be made by using the letters of the word “SIMPLETON” taken all at a time? Ans. There are „9‟ different letters of the word “SIMPLETON” Number of Permutations taking all the letters at a time = 9P9 = 9! = 362880. Number of permutations of n-thing, taken all at a time, in which „P‟ are of one type, „q‟ of them are of second-type, „r‟ of them are of third-type, and rest are all different is given by :- n!/(p! x q! x r!)
  • 7. Example: In how many ways can the letters of the word “Pre- University” be arranged? 13!/(2! X 2! X 2!) Number of permutations of n-things, taken „r‟ at a time when each thing can be repeated r-times is given by = nr. Proof. Number of ways of filling-up first –place = n Since repetition is allowed, so Number of ways of filling-up second-place = n Number of ways of filling-up third-place Number of ways of filling-up r-th place = n Hence total number of ways in which first, second ----r th, places can be filled-up = n x n x n ------------- r factors. = nr Example: John has 8 friends. In how many ways can he invite one or more of them to dinner? Ans. John can select one or more than one of his 8 friends. => Required number of ways = 28 – 1= 255.
  • 8. (iv) Number of ways of selecting zero or more things from „n‟ identical things is given by :- n+1 Example: In how many ways, can zero or more letters be selected form the letters AAAAA? Ans. Number of ways of : Selecting zero 'A's = 1 Selecting one 'A's = 1 Selecting two 'A's =1 Selecting three 'A's = 1 Selecting four 'A's = 1 Selecting five 'A's = 1 => Required number of ways = 6 [5+1] (V) Number of ways of selecting one or more things from „p‟ identical things of one type „q‟ identical things of another type, „r‟ identical things of the third type and „n‟ different things is given by :- (p+1) (q+1) (r+1)2n – 1
  • 9. Example: Find the number of different choices that can be made from 3 apples, 4 bananas and 5 mangoes, if at least one fruit is to be chosen. Ans: Number of ways of selecting apples = (3+1) = 4 ways. Number of ways of selecting bananas = (4+1) = 5 ways. Number of ways of selecting mangoes = (5+1) = 6 ways. Total number of ways of selecting fruits = 4 x 5 x 6 But this includes, when no fruits i.e. zero fruits is selected => Number of ways of selecting at least one fruit = (4x5x6) -1 = 119 Note :- There was no fruit of a different type, hence here n=o => 2n = 20=1 (VI) Number of ways of selecting „r‟ things from „n‟ identical things is „1‟. Example: In how many ways 5 balls can be selected from „12‟ identical red balls? Ans. The balls are identical, total number of ways of selecting 5 balls = 1. Example: How many numbers of four digits can be formed with digits 1, 2, 3, 4 and 5?
  • 10. Ans. Here n = 5 [Number of digits] And r = 4 [ Number of places to be filled-up] 5 Required number is P4 = 5!/1! = 5 x 4 x 3 x 2 x 1 Example: A child has 3 pocket and 4 coins. In how many ways can he put the coins in his pocket. Ans. First coin can be put in 3 ways, similarly second, third and forth coins also can be put in 3 ways. So total number of ways = 3 x 3 x 3 x 3 = 34 = 81 So, we should really call this a "Permutation Lock"! Permutations There are basically two types of permutation: 1. Repetition is Allowed: such as the lock above. It could be "333". 2. No Repetition: for example the first three people in a running race. You can't be first and second.
  • 11. 1. Permutations with Repetition These are the easiest to calculate. When you have n things to choose from ... you have n choices each time! When choosing r of them, the permutations are: n × n × ... (r times) (In other words, there are n possibilities for the first choice, THEN there are n possibilites for the second choice, and so on, multplying each time.) Which is easier to write down using an exponent of r: n × n × ... (r times) = nr Example: in the lock above, there are 10 numbers to choose from (0,1,..9) and you choose 3 of them: 10 × 10 × ... (3 times) = 103 = 1,000 permutations So, the formula is simply: nr where n is the number of things to choose from, and you choose r of them (Repetition allowed, order matters)
  • 12. 2. Permutations without Repetition In this case, you have to reduce the number of available choices each time. For example, what order could 16 pool balls be in? After choosing, say, number "14" you can't choose it again. So, your first choice would have 16 possibilites, and your next choice would then have 15 possibilities, then 14, 13, etc. And the total permutations would be: 16 × 15 × 14 × 13 × ... = 20,922,789,888,000 But maybe you don't want to choose them all, just 3 of them, so that would be only: 16 × 15 × 14 = 3,360 In other words, there are 3,360 different ways that 3 pool balls could be selected out of 16 balls. But how do we write that mathematically? Answer: we use the "factorial function" The factorial function (symbol: !) just means to multiply a series of descending natural numbers. Examples: 4! = 4 × 3 × 2 × 1 = 24 7! = 7 × 6 × 5 × 4 × 3 × 2 × 1 = 5,040 1! = 1 Note: it is generally agreed that 0! = 1. It may seem funny that multiplying no numbers together gets you 1,
  • 13. but it helps simplify a lot of equations. So, if you wanted to select all of the billiard balls the permutations would be: 16! = 20,922,789,888,000 But if you wanted to select just 3, then you have to stop the multiplying after 14. How do you do that? There is a neat trick ... you divide by 13! ... 16 × 15 × 14 × 13 × 12 ... = 16 × 15 × 14 = 3,360 13 × 12 ... Do you see? 16! / 13! = 16 × 15 × 14 The formula is written: where n is the number of things to choose from, and you choose r of them (No repetition, order matters) Examples: Our "order of 3 out of 16 pool balls example" would be: 16! 16! 20,922,789,888,000 = = = 3,360 (16-3)! 13! 6,227,020,800 (which is just the same as: 16 × 15 × 14 = 3,360)
  • 14. How many ways can first and second place be awarded to 10 people? 10! 10! 3,628,800 = = = 90 (10-2)! 8! 40,320 (which is just the same as: 10 × 9 = 90) Notation Instead of writing the whole formula, people use different notations such as these: Example: P(10,2) = 90 Combinations There are also two types of combinations (remember the order does not matter now): 1. Repetition is Allowed: such as coins in your pocket (5,5,5,10,10) 2. No Repetition: such as lottery numbers (2,14,15,27,30,33) How many permutations of 3 different digits are there, chosen from the ten digits 0 to 9 inclusive? A 84 B 120 C
  • 15. 504 D 720 Answer :The number of permutations of 3 digits chosen from 10 is 10 P3 = 10 × 9 × 8 = 720 Jones is the Chairman of a committee. In how many ways can a committee of 5 be chosen from 10 people given that Jones must be one of them? A 126 B 252 C 495 D 3,024 Answer: Jones is already chosen, so we need to choose another 4 from 9. In choosing a committee, order doesn't matter; so we need the number of combinations of 4 people chosen from 9 = 9 C4 = 9!/(4!)(5!) = (9 × 8 × 7 × 6)/(4 × 3 × 2 × 1) = 3,024/24 = 126 A password consists of four different letters of the alphabet. How many different possible passwords are there? (Using pen and paper can help you learn.) A 426
  • 16. B 456,976 C 14,950 D 358,800 Answer: The number of permutations of 4 letters chosen from 26 is 26 P4 = 26 × 25 × 24 × 23 = 358,800 A password consists of two letters of the alphabet followed by three digits chosen from 0 to 9. Repeats are allowed. How many different possible passwords are there? (Don't worry if you get it wrong ... you can learn from your mistakes.) A 492,804 B 650,000 C 676,000 D 1,757,600 Answer: The number of ways of choosing the letters = 26 × 26 = 676 The number of ways of choosing the digits = 10 × 10 × 10 = 1,000 So the number of possible passwords = 676 × 1,000 = 676,000 An encyclopedia has eight volumes. In how many ways can the eight volumes be replaced on the shelf? (Use pen and paper to work out the answer.) A 8 B
  • 17. 5,040 C 40,320 D 88 Answer:Imagine there are 8 spots on the shelf. Replace the volumes one by one. The first volume to be replaced could go in any one of the eight spots. The second volume to be replaced could then go in any one of the seven remaining spots. The third volume to be replaced could then go in any one of the six remaining spots. etc So the total number of ways the eight volumes could be replaced = 8! =8×7×6×5×4×3×2×1 = 40,320 Assuming that any arrangement of letters forms a 'word', how many 'words' of any length can be formed from the letters of the word SQUARE? (No repeating of letters) (No guessing! Be sure of your answer.) A 82 B 720 C 1,956 D 9,331 Answer: The number of one letter 'words' = 6P1 = 6 The number of two letter 'words' = 6P2 = 6 × 5 = 30
  • 18. The number of three letter 'words' = 6P3 = 6 × 5 × 4 = 120 The number of four letter 'words' = 6P4 = 6 × 5 × 4 × 3 = 360 The number of five letter 'words' = 6P5 = 6 × 5 × 4 × 3 × 2 = 720 The number of six letter 'words' = 6P6 = 6! = 720 So the total number of possible 'words' = 6 + 30 + 120 + 360 + 720 + 720 = 1,956 16 teams enter a competition. They are divided up into four Pools (A, B, C and D) of four teams each. Every team plays one match against the other teams in its Pool. After the Pool matches are completed: • the winner of Pool A plays the second placed team of Pool B • the winner of Pool B plays the second placed team of Pool A • the winner of Pool C plays the second placed team of Pool D • the winner of Pool D plays the second placed team of Pool C The winners of these four matches then play semi-finals, and the winners of the semi-finals play in the final. How many matches are played altogether? (Using pen and paper can help you learn.) A 23 B 31 C 32 D 63 Answer:The number of matches played in each Pool = 4C2 = 4!/(2!2!) = (4 × 3)/(2 × 1) = 6
  • 19. So the total number of Pool matches = 4 × 6 = 24 The winners and second placed teams play a further 4 matches. Then there are 2 semi-finals and 1 final So the total number of matches = 24 + 4 + 2 + 1 = 31 A restaurant offers 5 choices of appetizer, 10 choices of main meal and 4 choices of dessert. A customer can choose to eat just one course, or two different courses, or all three courses. Assuming all choices are available, how many different possible meals does the restaurant offer? A 329 B 310 C 200 D 19 Answer; A person who eats only an appetizer has 5 choices. A person who eats only a main meal has 10 choices. A person who eats only a dessert has 4 choices. A person who eats an appetizer and a main meal has 5 × 10 = 50 choices. A person who eats an appetizer and a dessert has 5 × 4 = 20 choices. A person who eats a main meal and a dessert has 10 × 4 = 40 choices. A person who eats all three courses has 5 × 10 × 4 = 200 choices So the total number of possible meals = 5 + 10 + 4 + 50 + 20 + 40 + 200 = 329
  • 20. Question: How many ways can 4 prizes be given away to 3 boys, if each boy is eligible for all the prizes? (1)256 (2)12 (3)81 (4) None of these Correct Answer - (3) Solution: Any one prize can be given to any one of the 3 boys and hence there are 3 ways of distributing each prize. Hence, the 4 prizes can be distributed in 34= 81 ways. Question: How many words of 4 consonants and 3 vowels can be made from 12 consonants and 4 vowels, if all the letters are different? (1)16C7.7! (2)12C4.4C3.7!
  • 21. (3)12C3.4C4 (4) 12C4 . 4C3 Correct Answer - (2) Solution: 12 4 consonants out of 12 can be selected in C4 ways. 3 vowels can be selected in 4C3 ways. Therefore, total number of groups each containing 4 consonants and 3 vowels = 12C4 . 4C3 Each group contains 7 letters, which can be arranging in 7! ways. 12 Therefore required number of words = 4 . 4C3 . 7! Question: In how many ways can the letters of the word EDUCATION be rearranged so that the relative position of the vowels and consonants remain the same as in the word EDUCATION? (1)9!/4 (2)9!/(4!.5!) (3)4!.5! (4) None of these Correct Answer - (3)
  • 22. Solution: The word EDUCATION is a 9 letter word, with none of the letters repeating. The vowels occupy 3, 5, 7th and 8th position in the word and the remaining 5 positions are occupied by consonants As the relative position of the vowels and consonants in any arrangement should remain the same as in the word EDUCATION, the vowels can occupy only the aforementioned 4 places and the consonants can occupy 1st, 2nd, 4th, 6th and 9th positions. The 4 vowels can be arranged in the 3rd, 5th, 7th and 8th position in 4! Ways. Similarly, the 5 consonants can be arranged in 1st, 2nd, 4th, 6th and 9th position in 5! Ways. Hence, the total number of ways = 4! . 5!. Question: There are 12 yes or no questions. How many ways can these be answered? (1)1024 (2)2048 (3) 4096 (4) 144 Correct Answer - (3)
  • 23. Solution: Each of the questions can be answered in 2 ways (yes or no) Therefore, no. of ways of answering 12 questions = 212 = 4096 ways. Question: What is the value of 1.1! + 2.2! + 3!.3! + ............ n.n!, where n! means n factorial or n(n-1)(n-2)...1 Solution: 1.1! = (2 -1).1! = 2.1! – 1.1! = 2! - 1! 2.2! = (3 - 1).2! = 3.2! - 2! = 3! - 2! 3.3! = (4 - 1).3! = 4.3! - 3! = 4! - 3! .. .. .. n.n! = (n+1 - 1).n! = (n+1)(n!) - n! = (n+1)! - n! Summing up all these terms, we get (n+1)! - 1! Question: In how many ways can the letters of the word MANAGEMENT be rearranged so that the two As do not appear together? Solution: The word MANAGEMENT is a 10 letter word. Normally, any 10 letter word can be rearranged in 10! ways. However, as there are certain letters of the word repeating, we need to account for those. In this case, the letters A, M, E and N repeat twice each.
  • 24. Therefore, the number of ways in which the letters of the word MANAGEMENT can be rearranged reduces to . The problem requires us to find out the number of outcomes in which the two As do not appear together. The number of outcomes in which the two As appear together can be found out by considering the two As as one single letter. Therefore, there will now be only 9 letters of which three of them E, N and M repeat twice. So these 9 letters with 3 of them repeating twice can be rearranged in ways. Therefore, the required answer in which the two As do not appear next to each other = Total number of outcomes - the number of outcomes in which the 2 As appear together => ways. **Question: There are 5 Rock songs, 6 Carnatic songs and 3 Indi pop songs. How many different albums can be formed using the above repertoire if the albums should contain at least 1 Rock song and 1 Carnatic song? (1)15624 (2)16384 (3)6144 (4) 240
  • 25. Correct Answer - (1) Solution: There are 2n ways of choosing ‘n’ objects. For e.g. if n = 3, then the three objects can be chosen in the following 23 ways - 3C0 ways of choosing none of the three, 3C1 ways of choosing one out of the three, 3C2 ways of choosing two out of the three and 3C3 ways of choosing all three. In the given problem, there are 5 Rock songs. We can choose them in 25 ways. However, as the problem states that the case where you do not choose a Rock song does not exist (at least one rock song has to be selected), it can be done in 25 - 1 = 32 - 1 = 31 ways. Similarly, the 6 Carnatic songs, choosing at least one, can be selected in 26 - 1 = 64 - 1 = 63 ways. And the 3 Indi pop can be selected in 23 = 8 ways. Here the option of not selecting even one Indi Pop is allowed. Therefore, the total number of combinations = 31 . 63 . 8 = 15624 Question: How many words can be formed by re-arranging the letters of the word ASCENT such that A and T occupy the first and last position respectively? (1)5! (2)4!
  • 26. (3)2! (4) 6! / 2! Correct Answer - (2) Solution: As A and T should occupy the first and last position, the first and last position can be filled in only one way. The remaining 4 positions can be filled in 4! Ways by the remaining words (S,C,E,N,T). hence by rearranging the letters of the word ASCENT we can form 1x4! = 4! words. **Question: A team of 8 students goes on an excursion, in two cars, of which one can seat 5 and the other only 4. In how many ways can they travel? Solution: There are 8 students and the maximum capacity of the cars together is 9. We may divide the 8 students as follows Case I: 5 students in the first car and 3 in the second Or Case II: 4 students in the first car and 4 in the second Hence, in Case I: 8 students are divided into groups of 5 and 3 in 8C3 ways. Similarly, in Case II: 8 students are divided into two groups of 4 and 4 in 8C4 ways.
  • 27. Therefore, the total number of ways in which 8 students can travel is 8C3 + 8C4 = 56 + 70 = 126. **Question: When four fair dice are rolled simultaneously, in how many outcomes will at least one of the dice show 3? Solution: When 4 dice are rolled simultaneously, there will be a total of 64 = 1296 outcomes. The number of outcomes in which none of the 4 dice show 3 will be 54 = 625 outcomes. Therefore, the number of outcomes in which at least one die will show 3 = 1296 – 625 = 671 Question: How many ways can 10 letters be posted in 5 post boxes, if each of the post boxes can take more than 10 letters? Solution: Each of the 10 letters can be posted in any of the 5 boxes. So, the first letter has 5 options, so does the second letter and so on and so forth for all of the 10 letters. i.e. 5.5.5….5 (upto 10 times) = 510. Here is another way to calculate it:
  • 28. Including "none" as an option, there are 6 choices of appetizer, 11 choices of main meal and 5 choices of dessert. Thus the total number of choices is 6x11x5=330. One of these is not a meal though (no appetizer, no main meal and no dessert), so there are 329 possible meals. Circular permutations There are two cases of circular-permutations:- (a) If clockwise and anti clock-wise orders are different, then total number of circular-permutations is given by (n- 1)! (b) If clock-wise and anti-clock-wise orders are taken as not different, then total number of circular-permutations is given by (n-1)!/2! Proof(a): (a) Let‟s consider that 4 persons A,B,C, and D are sitting around a round table Shifting A, B, C, D, one position in anticlock-wise direction, we get the following agreements:-
  • 29. Thus, we use that if 4 persons are sitting at a round table, then they can be shifted four times, but these four arrangements will be the same, because the sequence of A, B, C, D, is same. But if A, B, C, D, are sitting in a row, and they are shifted, then the four linear-arrangement will be different. Hence if we have „4‟ things, then for each circular-arrangement number of linear-arrangements =4 Similarly, if we have „n‟ things, then for each circular – agreement, number of linear – arrangement = n. Let the total circular arrangement = p Total number of linear–arrangements = n.p Total number of linear–arrangements = n. (number of circular-arrangements) Or Number of circular-arrangements = 1 (number of linear arrangements) n = 1( n!)/n circular permutation = (n-1)! Proof (b) When clock-wise and anti-clock wise arrangements are not different, then observation can be made from both sides, and this will be the same. Here two permutations will be counted as one. So total permutations will be half, hence in this case. Circular–permutations = (n-1)!/2
  • 30. Note: Number of circular-permutations of „n‟ different things taken „r‟ at a time:- (a) If clock-wise and anti-clockwise orders are taken as different, then total number of circular-permutations = n Pr /r (b) If clock-wise and anti-clockwise orders are taken as not different, then total number of circular – permutation = n Pr/2r Example: How many necklace of 12 beads each can be made from 18 beads of different colours? Ans. Here clock-wise and anti-clockwise arrangement s are same. 18 Hence total number of circular–permutations: P12/2x12 = 18!/(6 x 24) Restricted – Permutations (a) Number of permutations of „n‟ things, taken „r‟ at a time, when a particular thing is to be always included in each arrangement = r n-1 Pr-1 (b) Number of permutations of „n‟ things, taken „r‟ at a time, when a particular thing is fixed: = n-1 Pr-1 (c) Number of permutations of „n‟ things, taken „r‟ at a time, when a particular thing is never taken: = n-1 Pr.
  • 31. (d) Number of permutations of „n‟ things, taken „r‟ at a time, when „m‟ specified things always come together = m! x ( n- m+1) ! (e) Number of permutations of „n‟ things, taken all at a time, when „m‟ specified things always come together = n ! - [ m! x (n-m+1)! ] Example: How many words can be formed with the letters of the word „OMEGA‟ when: (i) „O‟ and „A‟ occupying end places. (ii) „E‟ being always in the middle (iii) Vowels occupying odd-places (iv) Vowels being never together. Ans. (i) When „O‟ and „A‟ occupying end-places => M.E.G. (OA) Here (OA) are fixed, hence M, E, G can be arranged in 3! ways But (O,A) can be arranged themselves is 2! ways. => Total number of words = 3! x 2! = 12 ways.
  • 32. ii) When „E‟ is fixed in the middle => O.M.(E), G.A. Hence four-letter O.M.G.A. can be arranged in 4! i.e 24 ways. (iii) Three vowels (O,E,A,) can be arranged in the odd- places (1st, 3rd and 5th) = 3! ways. And two consonants (M,G,) can be arranged in the even- place (2nd, 4th) = 2 ! ways => Total number of ways= 3! x 2! = 12 ways. (iv) Total number of words = 5! = 120! If all the vowels come together, then we have: (O.E.A.), M,G These can be arranged in 3! ways. But (O,E.A.) can be arranged themselves in 3! ways. => Number of ways, when vowels come-together = 3! x 3! = 36 ways => Number of ways, when vowels being never-together = 120-36 = 84 ways.
  • 33. Number of Combination of „n‟ different things, taken „r‟ at a time is given by:- n Cr= n! / r ! x (n-r)! Proof: Each combination consists of „r‟ different things, which can be arranged among themselves in r! ways. => For one combination of „r‟ different things, number of arrangements = r! For nCr combination number of arrangements: r n Cr n => Total number of permutations = r! Cr ---------------(1) But number of permutation of „n‟ different things, taken „r‟ at a time = nPr -------(2) From (1) and (2) : n Pr = r! . nCr or n!/(n-r)! = r! . nCr n or Cr = n!/r!x(n-r)! Note: nCr = nCn-r n or Cr = n!/r!x(n-r)! and nCn-r = n!/(n-r)!x(n-(n-r))!
  • 34. = n!/(n-r)!xr! Restricted – Combinations (a) Number of combinations of „n‟ different things taken „r‟ at a time, when „p‟ particular things are always included = n-p Cr-p. (b) Number of combination of „n‟ different things, taken „r‟ at a time, when „p‟ particular things are always to be excluded = n-pCr Example: In how many ways can a cricket-eleven be chosen out of 15 players? if (i) A particular player is always chosen, (ii) A particular is never chosen. Ans: (i) A particular player is always chosen, it means that 10 players are selected out of the remaining 14 players. 14 =. Required number of ways = C10 = 14C4 = 14!/4!x19! = 1365 (ii) A particular players is never chosen, it means that 11 players are selected out of 14 players.
  • 35. 14 => Required number of ways = C11 = 14!/11!x3! = 364 (iii) Number of ways of selecting zero or more things from „n‟ different things is given by:- 2n-1 Proof: Number of ways of selecting one thing, out of n-things = nC1 Number of selecting two things, out of n-things =nC2 Number of ways of selecting three things, out of n-things =nC3 Number of ways of selecting „n‟ things out of „n‟ things = nCn =>Total number of ways of selecting one or more things out of n different things = nC1 + nC2 + nC3 + ------------- + nCn = (nC0 + nC1 + -----------------nCn) - nC0 = 2n – 1 [ nC0=1] 1. Examples: 1. Suppose we want to select two out of three boys A, B, C. Then, possible selections are AB, BC and CA. Note: AB and BA represent the same selection. 2. All the combinations formed by a, b, c taking ab, bc, ca. 3. The only combination that can be formed of three letters a, b, c taken all at a time is abc.
  • 36. 4. Various groups of 2 out of four persons A, B, C, D are: AB, AC, AD, BC, BD, CD. 5. Note that ab ba are two different permutations but they represent the same combination. 2. Number of Combinations: The number of all combinations of n things, taken r at a time is: n n! n(n - 1)(n - 2) ... to r factors Cr = = . (r!)(n - r!) r! Note: n i. Cn = 1 and nC0 = 1. n ii. Cr = nC(n - r) Examples: 11 (11 x 10 x 9 x 8) i. C4 = = 330. (4 x 3 x 2 x 1) 16 x 15 x 14 16 x 15 x 14 ii. 16C13 = 16C(16 - 13) = 16C3 = = = 560. 3! 3x2x1 1. From a group of 7 men and 6 women, five persons are to be selected to form a committee so that at least 3 men are there on the committee. In how many ways can it be done? A.564 B. 645 C. 735 D.756 2. In how many different ways can the letters of the word 'LEADING' be arranged in such a way that the vowels always come together? A.360 B. 480 5040 C. 720 D.
  • 37. 3 how many different ways can the letters of the word In .'CORPORATION' be arranged so that the vowels always come together? A B 810 1440 . . C D 2880 50400 . . 4 Out of 7 consonants and 4 vowels, how many words of 3 consonants and 2 vowels can be formed? A B 210 1050 . . C D 25200 21400 . . 5. In how many ways can the letters of the word 'LEADER' be arranged? A B 72 144 . . C D 360 720 . . 6 In a group of 6 boys and 4 girls, four children are to be selected. In how many different ways can they be selected such that at least one boy should be there? A B 159 194 . . C D 205 209 . . E None of these . 7 How many 3-digit numbers can be formed from the digits 2, 3, 5, 6, 7 and 9, which are divisible by 5 and none of the digits is repeated? A B 5 10 . . C D 15 20 . . 8 In how many ways a committee, consisting of 5 men and 6 women can be formed from 8 men and 10 women? A 266 B 5040
  • 38. . . C D 11760 86400 . . E None of these . 9 A box contains 2 white balls, 3 black balls and 4 red balls. In how many ways can 3 balls be drawn from the box, if at least one black ball is to be included in the draw? A B 32 48 . . C D 64 96 . . E None of these . 10 In how many different ways can the letters of the word 'DETAIL' be arranged in such a way that the vowels occupy only the odd positions? A B 32 48 . . C D 36 60 . . E 120 . 11 In how many ways can a group of 5 men and 2 women be made out of a total of 7 men and 3 women? A B 63 90 . . C D 126 45 . . E 135 . 12 How many 4-letter words with or without meaning, can be formed out of the letters of the word, 'LOGARITHMS', if repetition of letters is not allowed? A B 40 400 . . C 5040 D 2520
  • 39. . . 13 In how many different ways can the letters of the word 'MATHEMATICS' be arranged so that the vowels always come together? A B 10080 4989600 . . C D 120960 None of these . . 14 In how many different ways can the letters of the word 'OPTICAL' be arranged so that the vowels always come together? A B 120 720 . . C D 4320 2160 . . E None of these . Answer:1 Option D Explanation: We may have (3 men and 2 women) or (4 men and 1 woman) or (5 men only). Required number of ways= (7C3 x 6C2) + (7C4 x 6C1) + (7C5) 7x6x5 6x5 = x + (7C3 x 6C1) + (7C2) 3x2x1 2x1 7x6x5 7x6 = 525 + x6 + 3x2x1 2x1 = (525 + 210 + 21) = 756. Answer:2 Option C Explanation:
  • 40. The word 'LEADING' has 7 different letters. When the vowels EAI are always together, they can be supposed to form one letter. Then, we have to arrange the letters LNDG (EAI). Now, 5 letters can be arranged in 5! = 120 ways. The vowels (EAI) can be arranged among themselves in 3! = 6 ways. Required number of ways = (120 x 6) = 720. Answer: 3 Option D Explanation: In the word 'CORPORATION', we treat the vowels OOAIO as one letter. Thus, we have CRPRTN (OOAIO). This has 7 (6 + 1) letters of which R occurs 2 times and the rest are different. 7! Number of ways arranging these letters = = 2520. 2! Now, 5 vowels in which O occurs 3 times and the rest are different, can be arranged 5! in = 20 ways. 3! Required number of ways = (2520 x 20) = 50400 Answer:4 Option C Explanation:
  • 41. Number of ways of selecting (3 consonants out of 7) and (2 vowels out of 4) = (7C3 x 4C2) 7x6x5 4x3 = x 3x2x1 2x1 = 210. Number of groups, each having 3 consonants and 2 vowels = 210. Each group contains 5 letters. Number of ways of arranging = 5! 5 letters among themselves =5x4x3x2x1 = 120. Required number of ways = (210 x 120) = 25200. Answer: 5Option C Explanation: The word 'LEADER' contains 6 letters, namely 1L, 2E, 1A, 1D and 1R. Required number of ways =6!= Answer:6 Option D Explanation: We may have (1 boy and 3 girls) or (2 boys and 2 girls) or (3 boys and 1 girl) or (4 boys). Required number 6 = ( C1 x 4C3) + (6C2 x 4C2) + (6C3 x 4C1) + (6C4) of ways = (6C1 x 4C1) + (6C2 x 4C2) + (6C3 x 4C1) + (6C2) 6x5 4x3 6x5x4 6x5 = (6 x 4) x + x4 + 2x1 2x1 3x2x1 2x1 = (24 + 90 + 80 + 15)
  • 42. = 209. Answer:7 Option D Explanation: Since each desired number is divisible by 5, so we must have 5 at the unit place. So, there is 1 way of doing it. The tens place can now be filled by any of the remaining 5 digits (2, 3, 6, 7, 9). So, there are 5 ways of filling the tens place. The hundreds place can now be filled by any of the remaining 4 digits. So, there are 4 ways of filling it. Required number of numbers = (1 x 5 x 4) = 20 Answer:8 Option C Explanation: Required number of ways= (8C5 x 10C6) = (8C3 x 10C4) 8 x 7 x 6 10 x 9 x 8 x 7 = 3 x 2 x 1x 4 x 3 x 2 x 1 = 11760. Answer9: Option C Explanation: We may have(1 black and 2 non-black) or (2 black and 1 non-black) or (3 black). Required number of ways= (3C1 x 6C2) + (3C2 x 6C1) + (3C3) 6x5 3x2 = 3x + x6 +1 2x1 2x1 = (45 + 18 + 1) = 64.
  • 43. Answer:10 Option C Explanation: There are 6 letters in the given word, out of which there are 3 vowels and 3 consonants. Let us mark these positions as under: (1) (2) (3) (4) (5) (6) Now, 3 vowels can be placed at any of the three places out 4, marked 1, 3, 5. Number of ways of arranging the vowels = 3P3 = 3! = 6. Also, the 3 consonants can be arranged at the remaining 3 positions. Number of ways of these arrangements = 3P3 3! = 6. Answer:11 Option A Explanation: 7x 7 3 7 3 Required number of ways = ( C5 x C2) = ( C2 x C1) 6 x = = 2x 3 63. 1 Answer:12 Option C Explanation: 'LOGARITHMS' contains 10 different letters. Required number of = Number of arrangements of 10 letters, taking words 4 at a time. = 10P4 = (10 x 9 x 8 x 7) = 5040.
  • 44. Answer:13 Option C Explanation: In the word 'MATHEMATICS', we treat the vowels AEAI as one letter. Thus, we have MTHMTCS (AEAI). Now, we have to arrange 8 letters, out of which M occurs twice, T occurs twice and the rest are different. 8! Number of ways of arranging these letters = = 10080. (2!)(2!) Now, AEAI has 4 letters in which A occurs 2 times and the rest are different. 4! Number of ways of arranging these letters = = 12. 2! Required number of words = (10080 x 12) = 120960 Answer:14 Option B Explanation: The word 'OPTICAL' contains 7 different letters. When the vowels OIA are always together, they can be supposed to form one letter. Then, we have to arrange the letters PTCL (OIA). Now, 5 letters can be arranged in 5! = 120 ways. The vowels (OIA) can be arranged among themselves in 3! = 6 ways. Required number of ways = (120 x 6) = 720.
  • 45. ASSIGNMENT Question 1 The principal wants to arrange 5 students on the platform such that the boy SALIM occupies the second position and such that the girl SITA is always adjacent to the girl RITA . How many such arrangements are possible? Question 2 When a group- photograph is taken, all the seven teachers should be in the first row and all the twenty students should be in the second row. If the two corners of the second row are reserved for the two tallest students, interchangeable only b/w them, and if the middle seat of the front row is reserved for the principal, how many such arrangements are possible? Question 3 If there are six periods in each working day of a school, in how many ways can one arrange 5 subjects such that each subject is allotted at least one period? Question 4 How many numbers greater than 4,00,000 can be formed by using the digits 0, 2, 2, 4, 4, 5? **Question 5 The letters of the word ZENITH are written in all possible order. How many words are possible if all these words are written out as in a dictionary? What is the rank of the word ZENITH? **Question 6 How many natural numbers not exceeding 4321 can be formed with the digits 1,2,3 and 4 if the digits can repeat? **Question 7 Three married couples are to be seated in a row having six seats in a cinema hall. If spouses are to be seated next to each other, in how many ways can they be seated? Find also the number of ways of their seating if all the ladies sit together. Question8 A boy has three library tickets and 8 books of his interest in the library. Of these 8, he does not want to borrow Chem. part II, unless Chem. Par I is also borrowed. In how many ways can be choose the three books to be borrowed? Question 9 A polygon has 44 diagonals. Find the number of its sides. ** Question10 In how many ways can 10 things be equally divided b/w (i) two persons (ii) two heaps ? **Question11 In an exam. there are three multiple choice questions and each question has 4 choices. Find the no. Of ways in which a student fails to get all answer correct.
  • 46. Question12 In how many ways can the letters of the word PERMUTATIONS be arranged if the (i) Words start with P and end with S? (ii) Vowels are all together ? (iii) T’s are together? (iv) There is no restriction? (v) There are always 4 letters b/w P & S? Question 13 Find the no. Of perm. Of n things taken r at a time in which 2 given things always occur. In how many pers. , they are always excluded? **Question 14 There are 10 points in a plane , no three of which are in the same st. Line excepting 4 points, which are collinear. Find the (i) No. Of st. Lines obtained from the pairs of these points , (ii) no. Of ∆ that can be formed with the vertices as these points. Question 15 Find the no. of per. Of n different things taken r at a time such that two specific things occur together. ANSWERS ( WITH HINTS) Answer 1 since the boy SALIM occupies the second position, we have to arrange the remaining 4 students, according to condition two seats III, IV OR IV , V May be occupied by SITA & RITA in 4 ways, then remaining seat may be occupied by 5th student in 1 way only.So no. Of arrangements = 2.4.1=8. Answer 2 The remaining 6 teachers can be arranged in the front row in 6! Ways. (∵ middle seat is reserved for principal). The remaining 18 students can be arranged in the second row in 18! Ways. (two corner are reserved for 2 tallest , they can be occupied 2 ways) ∴ total no. Of ways = 6!.(18)!.2! Answer 3 Five subjects can be allotted 5 periods out of the six periods in 6P5 ways. Now one period is left and it can be allotted to any one of the 5 subjects in 5 ways. So, total no. Of ways = 6P5 . 5 = 3600. Answer 4 When 5 occurs the extreme left position(greater than 4) , then no. Of ways = = 30. When 4 occurs at extreme left , then
  • 47. no. Of ways = 5!/2!=60 ( 0,2,2,4,5[2 appears twice]) ∴ total no. Of ways 90. Answer 5 (i) total no. Of possible words = 6!=720 (ii) The no. Of words beginning with E = 5! = 120 , similarly for H ,I, N or T is 120. The words start with Z will also be 120 and will have their rank from 601 to 720.Of these 120 words start with Z, the no. Of words with E in the second place is 120/5 = 24. ZE**** will have their rank from 601 to 624. Of these 24 words , NO. Of words with H(or I or N or T) in the third place is 24/2=6 Rank order : ZEH*** , ZEI***, ZEN*** 601 to 606, 607 to 612, 613 to 618 resp., start with ZEN are 613...ZENHIT, 614... ZENHTI, 615... ZENIHT, 616... ZENITH. Answer 6 1- digit nos. = 4, 2-digit nos. = 4x16, 3-digit nos. = 64, 4-digit nos. 256. Out of 256 , let us discard those which are greater than 4321. (i) 4 at thousand’s place & hundred’s place= 1.1.4.4=16 , 4 4 1 1 2 2 3 3 4 4 Similarly (ii) 4 at thousand’s place ,3 at hundred’s place, 3 or 4 in ten’s = 1.1.2.4=8, (iii) 4 at thousand’s place ,3 at hundred’s place, 2 in ten’s & 2,3,4 at unit’s= 1.1.1.3=3. Total 4-digit no.greater than 4321 are = 27, answer is 256-27=313. Answer 7(i) Let A, B, C be three married couples can be seated = 3! & can sit in 2 ways =2! ∴ req. Ways of seating = 3!.2!.2!.2!=48 . (ii) three ladies can be seated in 4 ways which are at seat number (1,2,3), (2,3,4), (3,4,5), (4,5,6). They can interchange their seats =3! Ways. Men can be seated at three remaining seats in 3! , so req. No. Of ways = 4.3!.3!= 144 . Answer 8 case-1 he borrows chem.. part II, chem.. part I and one more book out of the remaining 6 (8-2) books.
  • 48. Case-2 he does not borrow chem.. part-II and , so borrow all 3 books out of 7 ∴ total no. Of ways = 6C1 + 7C3=41. Answer 9 No. Of diagonals = nC2 - n = 44 ( nC2 = no.of st. Lines of polygon n sides). Answer 10 (i) 10 things be equally divided b/w two persons, groups are distinct , 10!/ (5!)2 = 252 (ii) 10 things be equally divided b/w two heaps no distinction can be made = ½ (252)=126. Answer 11 Req. No. Of ways= 4.4.4 – 1=63. Answer 13 (i) 10!/2! (ii) consider 5 vowels as one letter (8!/2!) .(5!) (iii) consider 2 T’s as one letter = (11)! (iv) 12!/2! (v) 12 letters in 12 places as P****S****** leaving 4 places in between , Thus P & S may be filled up in 7 ways, same for S & P , remaining 10 letters are arranged in 10!/2! Ways ∴ req. No. of ways 10!/(3!.2!.2!) = 151200. Answer 14 No. Of per. Of n things taken r at a time is same as the no. Of ways of filling up r places with n things, we arrange 2 things which can be arranged in r places in rP2 ways. Also remaining (r-2) places can be filled up with remaining (n-2) things in n-2Pr-2 ways. No. Of per. (2 are included) = rp2 . n-2Pr-2 , No. Of per. (2 are not included) = n-2Pr Answer 15 (i) No. Of st. Lines formed by joining 10 points , taking 2 at a time = 10C2 = 45. No. Of st. Lines joining 4 points = 4 C2 = 6 , but 4 are collinear , when join pairwise give only one line. Req. No. of lines = 45 – 6 + 1=40 (ii) No. of ∆s formed by joining the points , taken 3 at a time = 10 C3 = 120, no. of ∆s formed by joining 4 points , taken 3 at a time = 4C3 = 4 Req. No. of ∆s = 120 – 4 = 116. Answer 16 Two specific things can be in r places in (r-1) ways and two things can be arranged among themselves in 2! Ways and the remaining (n-2) things will be arranged in (r-2) places in n-2Pr-2 ways. Req. Per. = 2! (r-1) . n-2Pr-2 .